Test Bank – ACP: Respiratory Medicine

Asthma

1. A 42-year-old man with a history of wheezing and shortness of breath is referred to your pulmonary clinic for management of asthma. The diagnosis of asthma was apparently based on symptoms and evidence of obstruction on pulmonary function testing.

During an episode of airflow obstruction, which of the following findings would be specific for a diagnosis of asthma in this patient?
A. Depressed diffusing capacity of the lung for carbon monoxide (DLCO)
on pulmonary function testing
B. A normal alveolar-arterial difference in oxygen (A-aDO2) gradient
C. Improvement after administration of an inhaled bronchodilator
D. Improvement after administration of corticosteroids
E. The episode is associated with ingestion of a nonsteroidal antiinflammatory drug (NSAID)

Key Concept/Objective: To understand the differential diagnosis of asthma

DLCO may be elevated in some patients with asthma, possibly because of greater recruitment of capillaries from higher pulmonary arterial pressure. Depression of DLCO in the setting of pulmonary obstruction is characteristic of chronic obstructive pulmonary disease, not asthma. Because of ventilation-perfusion mismatching, an elevation in A-aDO2 is common, but severe hypoxemia is rare. A normal A-aDO2 gradient in the setting of obstruction and a finding of wheezes that are heard loudest over the neck and that are transmitted with less intensity to the lung periphery are suggestive of partial upper airway obstruction. Other factors that suggest upper airway obstruction are the patient’s perception that the problem is in the throat, hoarseness, a cough that sounds unusual, and a history of trauma, surgery, or prolonged intubation of the upper airway. Cardiac asthma may improve after administration of an inhaled bronchodilator. Therefore, one should exercise caution in making a diagnosis of bronchospasm on the basis of bronchodilator responsiveness, especially in the acute setting. Similarly, corticosteroids can occasionally relieve symptoms of upper airway obstruction by decreasing edema. However, bronchospasm that is associated with ingestion of aspirin or NSAIDs is highly suggestive of asthma. Perhaps 10% to 20% of patients with asthma exhibit an idiosyncratic reaction to aspirin and other NSAIDs. Within 15 minutes to 4 hours, patients may experience significant worsening of airflow obstruction and nasal or ocular symptoms. (Answer: E—The episode is associated with ingestion of a nonsteroidal anti-inflammatory drug [NSAID])

2. You are caring for a young woman with asthma who has symptoms almost on a daily basis. Although her symptoms occur at various times during the day, they occur more frequently at night. Currently, her medical regimen consists only of a short-acting inhaled beta-adrenergic agonist for rescue. The patient is trying to become pregnant.

Of the following, which is the best therapeutic step to take next for this patient?

A. A long-acting selective beta2-adrenergic agonist should be added to her regimen
B. A low-dose oral steroid should be added to her regimen
C. An inhaled corticosteroid should be added to her regimen
D. Theophylline should be added to her regimen
E. Ipratropium bromide should be added to her regimen

Key Concept/Objective: To understand the fundamental principles of the management of asthma

This patient has moderate asthma, and her current medical regimen is insufficient. An inhaled corticosteroid should be added to her regimen. Patients with daily symptoms of airflow obstruction should use inhaled steroids regularly. Long-acting selective beta2adrenergic agonists should not be used in place of low-dose inhaled steroids, but they can often be used to reduce (though not eliminate) steroid use. If taken before bedtime, they may be particularly helpful in patients with prominent nocturnal symptoms because of their long duration of action. However, their safety during pregnancy has not been thoroughly evaluated. Patients who remain symptomatic despite use of inhaled corticosteroids and a combination of inhaled beta-adrenergic agonist, theophylline, and possibly ipratropium bromide may require alternate-day oral steroid therapy at the minimal dosage necessary to maintain the patient’s symptoms at an acceptable level. Theophylline is less effective than the combination of a long-acting inhaled beta2-adrenergic agonist and inhaled corticosteroids. Anticholinergic agents such as ipratropium bromide have modest bronchodilatory effect in patients with asthma and could be added in refractory cases. The primary indication for anticholinergic agents is chronic obstructive pulmonary disease. Incidentally, this patient should probably be advised to wait until her asthma is better controlled before becoming pregnant. During pregnancy, approximately one third of patients experience improvement in their symptoms of asthma, and one third remain stable, but in one third, symptoms worsen. Those with more severe asthma are at greater risk of their symptoms worsening. (Answer: C— An inhaled corticosteroid should be added to her regimen)

3. A 42-year-old bakery worker presents with a complaint of cough and wheezing; he has been experiencing these symptoms for the past 2 months. He has been working at the bakery for the past 2 years. You consider occupational asthma in your differential diagnosis.

Which of the following statements accurately characterizes the evaluation and treatment of this patient?
A. Occupational asthma is unlikely because the patient was exposed to the work environment for almost 2 years before developing symptoms
B. Asthma that persists after the patient stops going to the workplace excludes occupational asthma as the diagnosis
C. Skin testing with a soluble extract of the suspected offending agent confirms a diagnosis of occupational asthma
D. If a diagnosis of occupational asthma is made, the patient should be advised to take an inhaled short-acting beta2-adrenergic agonist before and during work, as needed
E. Onset of symptoms hours after leaving the workplace supports a diagnosis of occupational asthma

Key Concept/Objective: To understand the diagnosis and treatment of occupational asthma

The typical history of a patient with occupational asthma is that after the patient has spent a few months (but sometimes up to several years) at a job, he or she experiences coughing, wheezing, and chest tightness shortly after arriving at the workplace. In most cases, occupational asthma is cured by removal of the offending agent or transfer of the patient from the site of the offending agent. Transfer of the patient to a job that merely reduces rather than eliminates exposure does not effectively relieve symptoms. Trying to treat occupational asthma with beta agonists without having the patient avoid exposure to the offending agent is not recommended. In a few cases, symptoms of occupational asthma continue for years after the patient has left the workplace. Skin testing with the appropriate soluble extracts assesses only for sensitization to the agent. Many workers exhibit positive results on skin testing but have no evidence of asthma. Some persons with occupational asthma report a delayed onset of asthmatic symptoms: symptoms begin hours after the patient leaves the workplace. This can make recognition of an association with an offending agent difficult. (Answer: E—Onset of symptoms hours after leaving the workplace supports a diagnosis of occupational asthma)

4. A 38-year-old woman with known long-standing asthma presents with cough, wheezing, and fever; chest x-ray reveals a right upper lobe infiltrate. The patient is admitted to the hospital. After several days of treatment with antibiotics, her symptoms do not improve, nor is improvement seen in the infiltrate. Her blood work reveals a normal white blood cell (WBC) count, but there is significant eosinophilia. You suspect the diagnosis of allergic bronchopulmonary aspergillosis (ABPA).

Which of the following statements regarding the diagnosis of this patient is false?

A. Chest x-ray characteristically shows central bronchiectasis
B. The disease rarely occurs in patients with asthma
C. Diagnostic criteria include eosinophilia, an elevation in total serum IgE level, a positive immediate skin-test reaction to Aspergillus antigen, and elevated levels of IgE and IgG antibodies specific to Aspergillus
D. The chronic form of the disease can mimic tuberculosis

Key Concept/Objective: To be able to recognize ABPA

ABPA is caused by a hypersensitivity reaction to the colonization of the airways by Aspergillus species. The acute form of the disease is characterized by fever, flulike symptoms, and myalgias; it is often confused with acute bacterial pneumonia. The presence of sputum and blood eosinophilia is highly characteristic; sputum cultures are negative for pathogenic bacteria. The chest x-ray is characterized by pulmonary infiltrates with corresponding atelectasis of the affected segment or lobe. These findings are related to the presence of tenacious secretions with obstruction of the bronchial airways. Migratory pulmonary infiltrates are not uncommon. The chronic form of the disease is characterized radiologically by central bronchiectasis and upper lobe infiltrates with corresponding volume loss that mimics tuberculosis. The diagnostic criteria for ABPA include (1) repeated growth of Aspergillus species in culture; (2) positive skin-test reaction to Aspergillus antigen; (3) elevated total serum IgE level (> 1,000 ng/ml in patients not receiving steroids); and (4) elevated levels of IgG and IgE antibodies specific against Aspergillus. Systemic steroids are used in treatment. New data suggest that the use of itraconazole in combination with inhaled steroids may be useful. This disease is seen almost exclusively in patients with long-standing asthma; occasionally, ABPA is diagnosed in patients with cystic fibrosis. (Answer: B—The disease rarely occurs in patients with asthma)

5. A 45-year-old man comes to your office to establish primary care. He has had asthma since childhood, and he has been experiencing occasional wheezing, shortness of breath, and cough productive of yellow sputum. He cannot identify specific irritants that trigger his asthma. He does not engage in regular exercise. He lives with his wife and their two children in an apartment building and works in an auto-body shop. He has no nasal polyps, and his physical examination is unremarkable.

Which of the following statements about this patient’s condition is most likely to be true?

A. Because he has no specific allergic precipitants (i.e., he has intrinsic asthma), he is more likely to respond to an inhaled steroid
B. His occasional cough associated with yellow sputum is likely to be infectious in origin
C. The fact that his physical examination is normal should raise suspicion that his symptoms are caused by something other than asthma alone
D. Occupational asthma is a strong possibility
E. You should caution him against starting a regular exercise program, because this may worsen his asthma symptoms

Key Concept/Objective: To understand that occupational asthma is common and that patients with long-standing asthma may not be aware that an occupational irritant is contributing to their asthma

Occupational exposure plays a role in 10% of patients with asthma. Patients with asthma often experience delayed hypersensitivity reactions more than 12 hours after exposure; because of this, a patient who had asthma before starting a job may not be aware of a noxious irritant in the workplace. Workers in auto-body shops are at risk for occupational asthma caused by paint spray. The distinction between intrinsic and extrinsic asthma has no bearing on asthma management. Eosinophils and their debris often cause yellow discoloration of sputum even in the absence of infection. Patients often have a normal physical examination between exacerbations. Although exercise can trigger asthma, with appropriate therapy almost all patients with asthma can perform regular exercise without difficulty. (Answer: D—Occupational asthma is a strong possibility)

6. At your urging, the patient in Question 5 attempts to start exercising, but he finds that he develops shortness of breath soon after he finishes jogging.

Which of the following statements about this patient is most correct?

A. His shortness of breath may very well result from being out of shape
B. He would benefit from use of ipratropium bromide before exercise
C. He would benefit from use of a beta agonist before exercise
D. He would benefit from use of theophylline before exercise
E. He would benefit from use of an inhaled steroid before exercise

Key Concept/Objective: To understand the appropriate therapy for exercise-induced asthma

The most effective therapy for exercise-induced asthma is an inhaled beta agonist. Cromolyn is also effective, and newer leukotriene modifiers may also have a role. Theophylline, corticosteroids, and anticholinergics have no role in the treatment of exercise-induced asthma. This patient’s symptoms are clearly not caused by deconditioning, because they occur only after he stops exercising. (Answer: C—He would benefit from use of a beta agonist before exercise)

7. The patient in Question 5 returns to clinic for follow-up 12 weeks later. He mentions that 7 days ago he had a headache, for which he took two aspirin. Later, as the headache began to subside, he also developed itchy eyes and an itchy throat.

Which of the following statements about this patient is most correct?

A. Because he does not have nasal polyps, it is unlikely that he has aspirin hypersensitivity
B. He is likely to have similar reactions to all NSAIDs
C. This reaction was the result of salicylate sensitivity, so ibuprofen should be safe for him to use
D. This reaction suggests that he would not benefit from a leukotriene modifier
E. A COX-2 inhibitor, such as celecoxib, is likely to be safe for him to use

Key Concept/Objective: To understand the relationship between asthma and aspirin hypersensitivity

Aspirin hypersensitivity can initially present with bronchoconstriction or other allergic symptoms. Cross-reactivity with other NSAIDs is almost universal, because the causal mechanism is likely mediated by COX. Therefore, all NSAIDs, including COX-2 inhibitors, are likely to cause a reaction. Despite the classic triad, asthma and aspirin hypersensitivity often coexist in the absence of nasal polyps. Leukotriene modifiers are likely to be more effective in patients with aspirin hypersensitivity because of their effect on the COX pathway. (Answer: B—He is likely to have similar reactions to all NSAIDs)

8. A 28-year-old woman seeks a second opinion for asthma that has been recently worsening. She has had asthma for the past 14 years, but over the past 6 months her symptoms have been more severe. In addition to wheezing, shortness of breath, and chest tightness, she has had intermittent fevers and flulike symptoms. She has been treated with multiple courses of antibiotics as well as increasing doses of inhaled steroids with no significant improvement. A chest x-ray shows patchy bilobar infiltrates, which are in different locations from those seen on a chest x-ray that she had 3 months ago. Serum Aspergillus serologies are very high.

Which of the following statements about this patient is false?

A. Her serum eosinophil count is probably elevated
B. A sputum culture for Aspergillus is likely to be positive
C. Any bronchial involvement is likely to be on the surface only, without tissue invasion
D. She may need to be treated with systemic corticosteroids
E. A trial of antifungal therapy will not be helpful

Key Concept/Objective: To understand the pathophysiology, diagnosis, and treatment of allergic bronchopulmonary aspergillosis

Allergic bronchopulmonary aspergillosis is a hypersensitivity reaction to colonization of the airways by Aspergillus. Recent studies have shown that the combination of the antifungal itraconazole and inhaled steroids may be effective treatment. If they fail, systemic corticosteroids may be necessary. Allergic bronchopulmonary aspergillosis typically causes an elevated serum eosinophil count, and sputum cultures will test positive. It is very unlikely that there will be fungal tissue invasion. (Answer: E—A trial of antifungal therapy will not be helpful)

For more information, see Staton GW, Ingram RH Jr: 14 Respiratory Medicine: II Asthma. ACP Medicine Online (www.acpmedicine.com). Dale DC, Federman DD, Eds. WebMD Inc., New York, August 2002

Chronic Obstructive Diseases of the Lung

9. A 57-year-old patient who smokes cigarettes presents with chronic productive cough and persistent progressive exercise limitation that is a result of breathlessness.

For this patient, which of the following statements is true?

A. Significant airway obstruction occurs in only 10% to 15% of people who smoke
B. The best tool for assessing the severity of obstruction is the ratio of forced expiratory volume in 1 second to forced vital capacity (FEV1/FVC)
C. Chronic bronchitis is a clinical diagnosis defined as the presence of cough and sputum production on most days for at least 3 consecutive months in a year
D. Measurement of lung volumes in patients with chronic airway obstruction (CAO) uniformly reveals an increased residual volume and a decreased functional residual capacity (FRC)

Key Concept/Objective: To understand the pathogenesis and pathophysiology of chronic obstructive pulmonary disease (COPD)

Chronic bronchitis and emphysema are by far the most common causes of chronic airflow obstruction. Chronic bronchitis is defined as the presence of cough and sputum on most days for at least 3 months of the year for a minimum of 2 years in succession. Emphysema is a destructive process involving the lung parenchyma and is defined in pathologic terms. Only 10% to 15% of smokers experience clinically significant airway obstruction. Although a low FEV1/FVC and a decrease in expiratory flow rates prove obstruction, the best measurement for assessing the severity of the obstruction is FEV1. Measurement of lung volumes uniformly reveals an increased residual volume (RV) and a normal to increased FRC. RV may be two to four times higher than normal because of slowing of expiratory flow and trapping of gas behind prematurely closed airways. FRC increases by two mechanisms: dynamic hyperinflation and activation of inspiratory muscles during exhalation. As a result, tidal breathing may take place at lung volumes as high as 1 to 2 L above normal levels. (Answer: A—Significant airway obstruction occurs in only 10% to 15% of people who smoke)

10. A 53-year-old man presents to establish primary care. He has a history of COPD and a 60 pack-year history of cigarette smoking. Currently, he smokes one pack of cigarettes a week. His COPD is currently managed with PRN albuterol administered with a metered-dose inhaler (MDI); a long-acting beta2 agonist; and an inhaled corticosteroid. The patient experiences dyspnea with moderate exertion; otherwise, he is functional. The results of a complete blood count (CBC) and serum chemistry are unremarkable. Pulse oximetry is significant for an O2 saturation of 96% on room air with no change after climbing and descending two flights of stairs. The patient says he would like to change his medications to nebulized bronchodilators. He also wonders which intervention is most likely to alter the natural history of his COPD.

For this patient, which of the following statements is true?
A. Long-term administration of oxygen will favorably alter the natural history of this patient’s disease
B. Probably the single most important intervention is to help this patient quit smoking
C. Physical training programs have been shown to significantly increase the exercise capacity of patients with even far-advanced chronic bronchitis and emphysema; such programs lead to objective improvements in lung function, as measured by FEV1
D. Nebulized bronchodilators are generally of greater benefit than
MDIs

Key Concept/Objective: To understand the importance and the benefits of smoking cessation in patients with COPD

Of the therapeutic measures available for patients with chronic bronchitis and emphysema, only smoking cessation and long-term administration of supplemental oxygen to the chronically hypoxemic patient have been definitively shown to alter the natural history of the disease favorably; in this patient with normal O2 saturation, administration of oxygen would be of no clinical benefit. Helping a patient to quit smoking is probably the single most important intervention; effective methods include counseling by physicians and nurses, use of nicotine replacement therapy, behavioral intervention (e.g., individual or group therapy), and several pharmacologic interventions (e.g., bupropion and nortriptyline). A variety of other therapies offer potential relief of symptoms in patients with COPD. These include the use of bronchodilators; anti-inflammatory therapy; administration of antibiotics during acute purulent exacerbations; pulmonary rehabilitation programs, including physical exercise and respiratory muscle training; and, for patients with cor pulmonale, the use of diuretics. There is no evidence that nebulized bronchodilators are of greater benefit than properly administered drypowder inhalers or MDIs used with a spacer. Physical-training programs, such as treadmill walking, significantly increase the exercise capacity of patients with even faradvanced chronic bronchitis and emphysema. These results have been achieved despite the fact that lung function, as reflected in such measurements as vital capacity and FEV1, is not affected and that maximal heart rate is generally not reached during the training sessions. (Answer: B—Probably the single most important intervention is to help this patient quit smoking)

11. A 62-year-old man with a history of COPD (FEV1, 38%) presents with worsening dyspnea, which now occurs at rest; purulent sputum; and wheezing of 6 days’ duration. He has increased the use of his inhalers without experiencing an improvement of symptoms. He denies having fever, chills, or pleuritic chest pain. A chest x-ray does not demonstrate an acute process. The patient is admitted for treatment of an acute exacerbation of COPD.

Which of the following statements regarding the management of acute exacerbations of COPD is true?
A. The duration of symptoms and the risk of serious deterioration in lung function can be reduced by at least a 14 to 21-day course of broad-spectrum antibiotics
B. The bronchodilator of choice in exacerbations of COPD is an anticholinergic such as ipratropium
C. Oxygen supplementation should be adjusted to maintain oxygen saturation at 95% or greater
D. In patients already receiving theophylline, measurement of the theophylline level is indicated because acute illness and some of the medications used to treat exacerbations can precipitate theophylline toxicity; however, there are no data that show that the addition of theophylline is beneficial for exacerbations of COPD

Key Concept/Objective: To understand the treatment of an acute exacerbation of COPD

Acute exacerbations are often precipitated by respiratory infection. Such infections may be caused by viral or bacterial pathogens. In patients with symptoms of infection, the duration of symptoms and the risk of serious deterioration in lung function can be reduced by a 7to 10-day course of broad-spectrum antibiotics. The bronchodilator of choice in exacerbations of COPD is a short-acting beta2 agonist such as albuterol, mainly because of its rapid onset of action. Once short-acting beta2-agonist therapy is started, inhaled anticholinergic therapy (e.g., ipratropium) should be initiated or increased. The dosage is usually 3 to 4 puffs but can be increased to 5 to 8 puffs every 3 to 4 hours. Oxygen supplementation should be adjusted to keep oxygen saturation around 90% to 92% to maintain tissue oxygenation while minimizing the risk of worsening hypercapnia. There are no data indicating that the addition of theophylline is beneficial for exacerbations of COPD. For patients already on theophylline, the theophylline level should be measured, because some of the medications used to treat COPD exacerbations can precipitate theophylline toxicity. (Answer: D—In patients already receiving theophylline, measurement of the theophylline level is indicated because acute illness and some of the medications used to treat exacerbations can precipitate theophylline toxicity; however, there are no data that show that the addition of theophylline is beneficial for exacerbations of COPD)

12. A 40-year-old man presents for evaluation of worsening dyspnea on exertion. He denies having a cough.
He has a remote history of tobacco use and a significant family history of emphysema. The results of a CBC and serum chemistry panel are unremarkable. A chest x-ray demonstrates attenuation of the pulmonary vasculature, predominantly in the lower lobes. Pulmonary function tests are consistent with severe airway obstruction.

Which of the following statements regarding a1-antitrypsin deficiency is true

A. Serious liver disease, usually in the form of cirrhosis, occurs in up to one third of adults with a1-antitrypsin deficiency
B. The mean age at onset of dyspnea is 55 to 60 years in nonsmokers and approximately 10 years earlier in those who smoke
C. In the United States, the prevalence of a1-antitrypsin deficiency caused by a homozygous PiZ genotype is one in 3,000 persons
D. The typical pathologic picture is a centriacinar emphysema with a basilar predominance

Key Concept/Objective: To understand the pathophysiology, epidemiology, and clinical presentation of a1-antitrypsin deficiency

In the United States, a1-antitrypsin deficiency caused by a homozygous PiZ genotype occurs in one in 3,000 persons; the prevalence is lower in African Americans. Emphysema develops in at least 80% of patients with homozygous PiZ a1-antitrypsin deficiency. The mean age at onset of dyspnea is 45 to 50 years in nonsmokers and approximately 10 years earlier in those who smoke. The typical pathologic picture is panacinar emphysema, but as many as 25% to 30% of nonsmoking patients and 60% of cigarette smokers report symptoms of chronic bronchitis as well. Many of the patients have evidence of enhanced airway reactivity. A significant incidence of bronchiectasis is detected by high-resolution CT scanning. Involvement of the lower lobes often predominates, perhaps because of increased neutrophil traffic and release of neutrophil elastase in the lower lung fields. The radiographic manifestation of this phenomenon is most commonly attenuation of the pulmonary vasculature to the lower lobes; in more advanced cases, basilar bullae may be seen. Serious liver disease, usually in the form of cirrhosis, occurs in 5% to 10% of adults with a1-antitrypsin deficiency and may provide a clue to the underlying enzyme deficiency in some patients. (Answer: C—In the United States, the prevalence of a1-antitrypsin deficiency caused by a homozygous PiZ genotype is one in 3,000 persons)

13. A 67-year-old man presents to your clinic for evaluation of dyspnea. He reports that his breathing has been worsening for years. He has a 100-pack-year history of cigarette smoking. His physical examination is notable for obesity, prolonged expiratory phase with faint wheezing, jugular veinous distention to the mandible, hepatosplenomegaly; and 2+ bilateral lower extremity edema.

Which of the following would NOT be characteristic of this patient with type B COPD?
A. Mild to moderate hypoxia with normal to slightly decreased arterial carbon dioxide tension (PaCO2)
B. Cough with sputum production
C. Progression to cor pulmonale; abnormal depression of arousal responses to hypoxia and hypercapnia during sleep
D. Increased resistance to airflow in both phases of the respiratory cycle

Key Concept/Objective: To know the clinical factors for differentiating and identifying COPD type A patients (pink puffers) from type B patients (blue bloaters)

Type A patients exhibit dyspnea with only mild to moderate hypoxemia (arterial oxygen tension [PaO2] levels are usually > 65 mm Hg) and maintain normal or even slightly reduced PaCO2 levels. These patients are sometimes referred to as pink puffers; they tend to be thin, to experience hyperinflation at total lung capacity, and to be free of signs of right heart failure. The pink puffer usually has severe emphysema. Type B patients have marked hypoxemia and peripheral edema resulting from right heart failure. These patients, who are sometimes called blue bloaters, typically exhibit cough and sputum production. They have frequent respiratory tract infections, experience chronic carbon dioxide retention (PaCO2 > 45 mm Hg), and have recurrent episodes of cor pulmonale. The blue bloater may also have pathologic evidence of severe emphysema; in addition, the blue bloater suffers from inflammation of large and small airways and possible defects in ventilatory control. (Answer: A—Mild to moderate hypoxia with normal to slightly decreased arterial carbon dioxide tension [PaCO2])

14. A 67-year-old man with a history of emphysema presents with a complaint of worsening dyspnea and cough that is productive of yellow-colored sputum. On pulmonary function testing, his FEV1 is 45%. Arterial blood gas measurements were performed several months ago. The baseline value for PaO2 was 53 mm Hg, and the carbon dioxide tension (PCO2) on room air was normal.

There is evidence showing improved survival for which of the following interventions (in addition to smoking cessation)?
A. Use of broad-spectrum antibiotics
B. Use of corticosteroids
C. Home oxygen therapy
D. Lung volume reduction surgery
E. Bronchodilator therapy

Key Concept/Objective: To understand that only smoking cessation and long-term administration of supplemental oxygen have been definitively shown to change the natural history of emphysema

Reviews and guidelines on the treatment of CAO have been published, but they disagree on recommendations. Suboptimal prescription of and adherence to appropriate therapies further complicate the management of CAO. Of the therapeutic measures available for patients with chronic bronchitis and emphysema, only smoking cessation and longterm administration of supplemental oxygen to the chronically hypoxemic patient have been definitively shown to favorably alter the natural history of the disease. A variety of other therapies offer potential relief of symptoms in patients with CAO. These include the use of bronchodilators; anti-inflammatory therapy; the administration of antibiotics during acute purulent exacerbations; pulmonary rehabilitation programs, including physical exercise and respiratory muscle training; and, for patients with cor pulmonale, the use of diuretics. A randomized, multicenter clinical trial comparing lung volume reduction surgery with continued medical treatment in 1,218 patients with severe emphysema found that the surgery increased the chance of improved exercise capacity but did not confer a survival advantage, except in patients who had both predominantly upper lobe emphysema and low exercise capacity after rehabilitation. Broad-spectrum antibiotics, corticosteroids, and bronchodilators help improve symptoms, not long-term survival. (Answer: C—Home oxygen therapy)

15. A 40-year-old man presents to your clinic for evaluation of dyspnea. The patient is a nonsmoker and reports a slow progression of breathlessness. He also reveals that several of his family members were diagnosed with emphysema early in life, but he is confused because they were nonsmokers.

Which of the following statements concerning a1-antitrypsin deficiency is true?
A. Emphysematous changes do not occur in the lower lobes of the lung
B. a1-Antitrypsin deficiency is not associated with cirrhotic liver disease
C. a1-Antitrypsin deficiency is not associated with a family history of emphysema in nonsmokers
D. Purified a1-antitrypsin is commercially available for treatment
E. a1-Antitrypsin levels higher than 40% of normal do not afford protection against the development of emphysema

Key Concept/Objective: To know the clinical characteristics of a1-antitrypsin deficiency and potential therapy to prevent disease progression

Emphysema develops in at least 80% of patients with homozygous PiZ a1-antitrypsin deficiency. The mean age at onset of dyspnea is 45 to 50 years in nonsmokers and approximately 10 years earlier in those who smoke. The typical pathologic picture is panacinar emphysema, but as many as 25% to 30% of nonsmoking patients and 60% of cigarette smokers report symptoms of chronic bronchitis as well. Many of the patients have evidence of enhanced airway reactivity. High-resolution CT scanning has detected a significant incidence of bronchiectasis in these patients. Involvement of the lower lobes often predominates, perhaps because of increased neutrophil traffic and the release of neutrophil elastase in the lower lung fields. The most common radiographic manifestation of this phenomenon is attenuation of the pulmonary vasculature to the lower lobes; in more advanced cases, basilar bullae may be seen. Features that would suggest a1-antitrypsin deficiency as the cause of a particular patient’s emphysema would thus include a family history of emphysema (especially among nonsmokers), the onset of symptoms at 30 to 50 years of age, the development of significant emphysema in a nonsmoker, and basilar predominance of the radiographic abnormalities. Serious liver disease, usually in the form of cirrhosis, occurs in 5% to 10% of adults with a1antitrypsin deficiency and may provide a clue to the underlying enzyme deficiency in some patients. It is thought that a1-antitrypsin levels higher than 40% of normal afford protection against the development of emphysema. For patients with homozygous PiZ deficiency, consideration should be given to administration of purified a1-antitrypsin, which is commercially available for replacement therapy. (Answer: D—Purified a1-antitrypsin is commercially available for treatment)

16. A 37-year-old woman is referred to you for evaluation of dyspnea, purulent cough, and recurrent pneumonia. The patient has a childhood history of recurrent pneumonia. She has no known contacts with persons with tuberculosis, and a test for the presence of purified protein derivative (PPD) is negative. She has smoked a pack of cigarettes each day for 15 years. Pulmonary function tests were interpreted as indicating mild airflow obstruction.

Which of the following features does NOT favor a diagnosis of bronchiectasis over a diagnosis of emphysematous lung disease in this patient?
A. Chronic cough and dyspnea without purulent sputum production
B. Tramlines noted on plain chest radiographs
C. Clinical improvement from broad-spectrum antibiotics and drainage
D. Massive hemoptysis
E. Clubbing of the digits

Key Concept/Objective: To understand that clubbing is a feature of bronchiectasis, not chronic airflow obstruction

Factors in the patient’s history, such as chronic cough and sputum purulence originating from a serious respiratory tract infection, often in childhood, strongly suggest the diagnosis of bronchiectasis. Bronchiectasis may be seen on the plain chest radiograph in a number of different patterns. Occasionally, the thickened walls of a dilated bronchus can be visualized as the bronchus courses with its longitudinal axis perpendicular to the x-ray beam. These parallel lines are approximately 1 mm thick and are referred to as tramlines. Airflow obstruction is generally the main abnormality seen on pulmonary function tests. The mainstays of therapy for bronchiectasis (including cystic fibrosis [CF] and primary ciliary dyskinesia), as for any chronic suppurative disease, are administration of antibiotics and drainage. Massive hemoptysis (200 ml of blood over a 24-hour period) can occur as a complication of bronchiectasis. Clubbing of the digits occurs in the majority of patients with significant bronchiectasis and is a valuable diagnostic clue, especially since clubbing of the digits is not a manifestation of CAO. (Answer: A—Chronic cough and dyspnea without purulent sputum production)

17. A 21-year-old woman with a history of CF who previously received health care services at a local children’s hospital now presents to your office to establish care as an adult.

Which of the following statements about CF is false

A. The median survival for women with CF is 28.3 years
B. The majority of patients with CF possess the ?F508 mutation, leading to an abnormal CF transmembrane regulator (CFTR)
C. Impaired clearance of secretions leads to recurrent pulmonary infections and bronchiectasis
D. Exocrine pancreatic function is maintained
E. Diagnosis can be made by the sweat chloride test or by genetic testing

Key Concept/Objective: To understand the diagnosis and clinical manifestations of CF

Although CF is an inherited disease that usually manifests itself in early childhood, a discussion of the condition in the context of general adult medicine is worthwhile for two reasons. First, increasing numbers of children with CF are now surviving into young adulthood: the median survival in the United States is 31.1 years in men and 28.3 years in women. Second, some patients have a variant form of the disease in which symptoms first appear during adolescence or adulthood. The genetic defects responsible for CF have been identified. The CF locus is on the long arm of chromosome 7, and it codes for a 1,480 amino acid polypeptide that has been named the CF transmembrane regulator (CFTR). In 70% of patients with CF, the 508th amino acid of this sequence is missing (?F508). It is likely that impaired tracheobronchial clearance of the abnormal secretions leads to widespread mucous plugging of airways, resulting in secondary bacterial infection, persistent inflammation, and consequent generalized bronchiectasis. Extrapulmonary manifestations may also suggest the diagnosis of CF. Prominent among these findings are pancreatic insufficiency with consequent steatorrhea, recurrent partial intestinal obstruction caused by abnormal fecal accumulation (the so-called meconium ileus equivalent), heat prostration, hepatic cirrhosis, and aspermia in men. The diagnosis can be established by abnormal results on a sweat test performed in a qualified laboratory using pilocarpine iontophoresis. In persons younger than 20 years, a sweat chloride level exceeding 60 mEq/L confirms the diagnosis; a value exceeding 80 mEq/L is required for diagnosis in persons 20 years of age or older. With the identification of the gene for CF, genetic screening has become available. (Answer: D—Exocrine pancreatic function is maintained)

18. A 53-year-old man with a 60-pack-year history of cigarette smoking presents with complaints of productive cough and dypsnea. He reports that for the past 3 months, he has been treated for bronchitis with antibiotics, but his symptoms have not resolved. Over the past several weeks, he has experienced progressive dypsnea on exertion. He denies having any chest discomfort or any other significant medical history. Currently, he is not taking any medications. His lung examination shows wheezing that resolves with expectoration of phlegm. Chest x-ray shows hyperinflation. Initial pulmonary function tests show the patient’s FEV1 to be 55% of the predicted value. Arterial blood gas measurements are as follows: PaO2, 75 mm Hg; alveolar carbon dioxide tension (PACO2), 55 mm Hg.

Which of the following is NOT true for this patient?

A. If this patient continues to smoke, his FEV1 value will continue to decrease two to three times faster than normal
B. If this patient stops smoking, the rate of decline in expiratory flow reverts to that of nonsmokers, and there may be a slight improvement in FEV1 during the first year
C. This patient would be expected to have evidence of extensive panacinar emphysema
D. This patient would be expected to have increased RV, increased FRC, and normal or increased total lung capacity (TLC)
E. This patient is at risk for right-sided heart failure

Key Concept/Objective: To understand the progression of chronic bronchitis and emphysema

Panacinar emphysema is common in patients with a1-antitrypsin deficiency. Centriacinar emphysema is commonly found in cigarette smokers and is rare in nonsmokers. Centriacinar emphysema is usually more extensive and severe in the upper lobes. In most cigarette smokers, a mixture of centriacinar and panacinar emphysema develops. In healthy nonsmokers, FEV1 begins declining at about 20 years of age and continues at an average rate of about 0.02 to 0.04 L/yr. In smokers with obstructive lung disease, FEV1 decreases, on average, two to three times faster than normal. When persons with mild to moderate airflow obstruction stop smoking, the rate of decline in expiratory flow reverts to that observed in nonsmokers, and there may be a slight improvement in FEV1 during the first year. Measurement of lung volumes uniformly reveals an increased RV and a normal to increased FRC. RV may be two to four times higher than normal because of slowing of expiratory flow and gas trapping behind prematurely closed airways. TLC is normal or increased. One group of patients (type A) exhibit dyspnea with only mild to moderate hypoxemia (PaO2 levels are usually > 65 mm Hg) and maintain normal or even slightly reduced PACO2 levels. They are sometimes referred to as pink puffers. The other clinical group of patients (type B) are sometimes called blue bloaters; they typically exhibit cough and sputum production, frequent respiratory tract infections, chronic carbon dioxide retention (PACO2 > 45 mm Hg), and recurrent episodes of cor pulmonale. In the type B patient, both alveolar hypoxia and acidosis (secondary to chronic hypercapnia) stimulate pulmonary arterial vasoconstriction, and hypoxemia stimulates erythrocytosis. Increased pulmonary vascular resistance, increased pulmonary blood volume, and, possibly, increased blood viscosity (resulting from secondary erythrocytosis) all contribute to pulmonary arterial hypertension. In response to long-term pulmonary hypertension, cor pulmonale generally develops: the right ventricle becomes hypertrophic, and cardiac output is increased by means of abnormally high right ventricular filling pressures. (Answer: C— This patient would be expected to have evidence of extensive panacinar emphysema)

19. A 43-year-old female patient with chronic bronchitis associated with a 40-pack-year history of cigarette smoking presents for a routine appointment. Although she has a productive cough on a daily basis, she denies having any dypsnea and is currently not taking any medication.

Which of the following measures will most alter the natural progression of this patient’s disease?
A. Daily bronchodilator use alone
B. Daily corticosteroid use alone
C. Daily prophylactic antibiotic
D. Daily pulmonary rehabilitation
E. Smoking cessation

Key Concept/Objective: To know key treatment measures for chronic bronchitis and emphysema

Of the therapeutic measures available for patients with chronic bronchitis and emphysema, only smoking cessation and long-term administration of supplemental oxygen to the chronically hypoxemic patient have been shown to alter the natural history of the disease favorably. Helping a patient to quit smoking is probably the single most important intervention. Most patients with chronic bronchitis and emphysema who are given a sufficiently strong bronchodilating medication will exhibit at least a 10% increase in maximal expiratory airflow. Dyspneic patients should be given a trial of bronchodilators even if pulmonary function testing shows that they do not manifest significant bronchodilation, because bronchodilator responsiveness may vary over time. Given the underlying pathophysiology of emphysema, corticosteroids would be expected to provide little benefit, because tissue destruction is the basic disease mechanism. Only some patients derive significant benefit from corticosteroids. Clinical trials of daily antibiotic use in patients with mild chronic airflow obstruction demonstrated that neither the degree of disability nor the rate of progression of disease was significantly altered by this intervention. Intermittent antibiotic administration is indicated for acute episodes of clinical worsening marked by increased dyspnea, excessive sputum production, and sputum purulence. Physical-training programs, such as treadmill walking, significantly increase the exercise capacity of patients with even far-advanced chronic bronchitis and emphysema. (Answer: E—Smoking cessation)

20. A 23-year-old male college student with no history of cigarette smoking presents with a complaint of productive cough that has not improved with three courses of antibiotics. He reports some intermittent wheezing and dyspnea, which have worsened over the past 2 days, but he has no fever. He states that he has had various recurrent respiratory infections ever since childhood. On examination, his chest xray shows diffuse increased markings with cystic spaces predominantly in the upper lobes and hyperinflation. Further testing reveals an abnormal sweat chloride test.

Which of the following is the most likely diagnosis for this patient?
A. Cystic fibrosis
B. Bronchiolitis obliterans
C. Asthma
D. a1-Antitrypsin deficiency

Key Concept/Objective: To understand the presentation of cystic fibrosis in adults

Although cystic fibrosis is an inherited disease that usually manifests itself in early childhood, increasing numbers of children with cystic fibrosis are now surviving into young adulthood, and some patients have a variant form of the disease in which symptoms first appear during adolescence or adulthood. The chest radiograph may strongly suggest the diagnosis of cystic fibrosis. The generalized bronchiectasis manifests itself as a diffuse increase in interstitial markings, and discrete bronchiectatic cysts are often visible; typically, involvement of the upper lobes predominates. The diagnosis can be established by abnormal results on a sweat test. Bronchiolitis is considered a disease of childhood. Bronchiolitis obliterans is a rare cause of chronic airflow obstruction in adults but can occur after inhalation of toxic gases (e.g., chlorine and nitrogen dioxide) and as an idiopathic phenomenon. Emphysema develops in at least 80% of patients with homozygous PiZ a1-antitrypsin deficiency. The mean age at onset of dyspnea is 45 to 50 years in nonsmokers and approximately 10 years earlier in those who smoke. (Answer: A—Cystic fibrosis)

21. A 56-year-old male industrial worker presents with concern of possible exposures that can cause lung disease. He has never smoked and is currently asymptomatic.

Which of the following diseases does NOT have an occupational exposure etiology?
A. Chronic bronchitis
B. Bronchiolitis obliterans
C. Bronchiectasis
D. Silo-filler’s disease

Key Concept/Objective: To know the risk factors and pathophysiology of bronchiectasis

Bronchiectasis is a chronic suppurative disease of the airways that if sufficiently widespread may cause chronic airflow obstruction. Bronchiectasis is a localized, irreversible bronchial dilatation caused by a destructive inflammatory process involving the bronchial walls. Necrotizing bacterial or mycobacterial infection is thought to be responsible for most cases of bronchiectasis. Adult-onset bronchiectasis may result from an untreated or inadequately treated bronchopneumonia that is caused by virulent organisms such as staphylococci or gram-negative bacilli. Prolonged exposure to respirable dusts in the work environment has long been recognized as a cause of so-called industrial or occupational bronchitis in nonsmoking workers engaged in occupations such as coal or gold mining, textile manufacturing, and cement and steel making. Bronchiolitis obliterans can occur with inhalation of toxic gases (e.g., chlorine and nitrogen dioxide). Silo-filler’s disease is an example of bronchiolitis obliterans resulting from toxic gas inhalation of nitrous oxides. (Answer: C—Bronchiectasis)

For more information, see Staton GW: 14 Respiratory Medicine: III Chronic Obstructive Diseases of the Lung. ACP Medicine Online (www.acpmedicine.com). Dale DC, Federman DD, Eds. WebMD Inc., New York, November 2004

Focal and Multifocal Lung Disease

22. A 31-year-old healthy man who has no significant medical history or current complaints presents to your office with concern about an abnormal chest x-ray that was taken at a local health fair. He has neither constitutional nor pulmonary symptoms. He reports no toxic exposures or family history of lung diseases. The physical examination is unremarkable. On review of the chest x-ray, a 1 cm focal lesion with central calcification is seen in right middle lobe. There are no previous films available for comparison.

Which of the following describes the most appropriate treatment plan for this patient?
A. Although asymptomatic, this patient requires thoracic surgery consultation and open lung biopsy
B. Given his young age and the appearance of the nodule, no further workup is necessary at this time; follow-up chest x-ray in 6 to 12 months is recommended
C. Bronchoscopy with airway inspection and likely transbronchial biopsy will probably yield a diagnosis; if bronchoscopy is unrevealing, the patient should be referred to thoracic surgery
D. Placement of purified protein derivative, examination of sputum for malignant cells (sputum cytology), and a high-resolution chest computed tomographic scan should be performed promptly, and the patient should be referred to thoracic surgery consultation for open lung biopsy

Key Concept/Objective: To understand the management of a single pulmonary nodule on chest x-ray

The management of a patient with a solitary lung nodule on chest x-ray can be challenging. In almost all cases, it should be assumed that the nodule is malignant. A benign etiology can be assumed if a chest radiograph taken 2 or more years earlier shows the lesion to have been the same size as or larger than it is currently. Such a situation could arise if the lesion went unrecognized on the initial film. Patients 35 years old or younger who are asymptomatic can also be managed in a conservative manner; otherwise, a thorough evaluation would be indicated. There are also classic benign patterns of calcification that obviate further assessment of single small nodules. For granulomas, such patterns include dense, perfectly central targets of calcium, ring calcification, and solid, dense calcification of the whole nodule. Given this patient’s young age and central calcification, follow-up chest x-ray is the most prudent approach to management. (Answer: B—Given his young age and the appearance of the nodule, no further workup is necessary at this time; follow-up chest x-ray in 6 to 12 months is recommended)

23. A 42-year-old white man whose medical history is unknown presents to the clinic with shortness of breath of new onset. He reports decreased appetite, malaise, and cough with minimal sputum. He also states that he feels warm at night. He has been almost completely bed-bound for the past week. Physical examination is unremarkable except that the patient appears older than his stated age and has decreased breath sounds in the left apex. A chest radiograph shows infiltrates on both the right and left apices; no cavitations are noted. There is also ipsilateral hilar adenopathy.

Which of the following statements is true regarding this patient’s chest radiograph?
A. Pneumococcal and other bacterial pneumonias can be ruled out, given the multifocal pattern of infiltrates
B. In light of the clinical presentation, reactivation of pulmonary tuberculosis seems likely; however, the lack of cavitations rules out this diagnosis
C. Given the vague complaints of this patient and the findings on chest radiography, the differential diagnosis should include bacterial pneumonia, reactivation tuberculosis, pulmonary thromboembolic disease, and sarcoidosis
D. Radiographic changes such as these, if caused by malignancy, are certainly metastatic and do not originate in the lung parenchyma

Key Concept/Objective: To understand the radiologic changes seen with tuberculosis and multifocal infiltrates

Most disorders that cause single infiltrates can also cause multiple infiltrates. For example, S. pneumoniae pneumonia and other bacterial pneumonias are occasionally multifocal; most viral pneumonias and pneumonias caused by Legionella species and Mycoplasma are commonly multifocal or diffuse. Pulmonary thromboembolism and sarcoidosis can also produce multifocal infiltrates. Reactivation tuberculosis is often multifocal. Bilateral infiltrates in the upper lung zones are most characteristic of reactivation tuberculosis. The upper lung zones are favored sites because a higher ratio of ventilation to perfusion results in higher local oxygen tension, which enhances growth of Mycobacterium tuberculosis. The apical and posterior segments of the upper lobes are most commonly involved, followed by the apical-posterior segments of the lower lobes. Cavitation is frequent, but even in the absence of cavitation, the diagnosis of tuberculosis should be considered when multifocal infiltrates are present. Alveolar cell carcinoma and Hodgkin disease usually present as focal infiltrates; however, they can also exhibit a pattern of multifocal infiltrates. Metastatic lesions to the lung are usually seen as ill-defined opacities without a lobar or segmental distribution. (Answer: C—Given the vague complaints of this patient and the findings on chest radiography, the differential diagnosis should include bacterial pneumonia, reactivation tuberculosis, pulmonary thromboembolic disease, and sarcoidosis)

24. A 43-year-old African-American woman who has had asthma for 16 years presents to your walk-in clinic with progressive dyspnea, chills, and productive cough. Physical examination reveals a thin woman in moderate distress. She is afebrile but has mild tachypnea and tachycardia. Lung examination reveals moderate air movement, diffuse wheezes, and egophony in the left upper lung zone without change in tactile fremitus. Chest radiography shows a segmental infiltrate of the left upper lobe with fingerlike shadows and dilated central bronchi.

Which of the following diagnoses best explains the constellation of clinical findings and radiologic changes?
A. Allergic bronchopulmonary aspergillosis
B. Alvelolar cell carcinoma with endobronchial invasion
C. Bronchiolitis obliterans organizing pneumonia (BOOP)
D. Caplan syndrome

Key Concept/Objective: To understand the differential diagnosis of a segmental infiltrate and the classic presentation of allergic bronchopulmonary aspergillosis

Allergic bronchopulmonary aspergillosis, which is also associated with asthma, is a hypersensitivity disease that primarily affects the central airways. Immediate and delayed hypersensitivity to Aspergillus are involved in the pathogenesis of this disorder. Onset of disease occurs most often in the fourth and fifth decades, and virtually all patients have long-standing atopic asthma. Even those few patients who do not have a history of documented asthma exhibit airflow obstruction when they present with this disorder. The typical patient has a long history of intermittent wheezing, after which the illness evolves into a more chronic and more highly symptomatic disorder with fever, chills, pulmonary infiltrates, and productive cough. The chest x-ray may show a segmental infiltrate or segmental atelectasis, most commonly in the upper lobes. Caplan syndrome is characterized by pulmonary nodules; it is seen exclusively in patients with rheumatoid arthritis. The constellation of long-standing asthma, wheezing on physical examination, and the presence of central dilated bronchi are not associated with either alveolar cell carcinoma or BOOP. In the patient with typical symptoms, the branching, fingerlike shadows from mucoid impaction of dilated central bronchi are pathognomonic of allergic bronchopulmonary aspergillosis. (Answer: A— Allergic bronchopulmonary aspergillosis)

For more information, see Ingram RH Jr: 14 Respiratory Medicine: IV Focal and Multifocal Lung Disease. ACP Medicine Online (www.acpmedicine.com). Dale DC, Federman DD, Eds. WebMD Inc., New York, December 2002

Chronic Diffuse Infiltrative Lung Disease

25. A 42-year-old African-American woman who works as a physician’s assistant presents to your office with progressive dyspnea and a nonproductive cough. After a careful history is obtained, no occupational or toxic exposures are readily identified. The patient is concerned that her symptoms are secondary to idiopathic pulmonary fibrosis (IPF). The physical examination is unremarkable. Chest radiography shows prominent hilar adenopathy with a diffuse interstitial process. The working diagnosis is sarcoidosis. You obtain the patient’s consent for a bronchoscopy and transbronchial biopsy, but she tells you she thinks an open lung biopsy is necessary for a diagnosis of sarcoidosis.

What is the correct response to this patient with regard to the appropriate workup of sarcoidosis?

A. “You are right, I will consult a thoracic surgeon.”
B. “You are right, but in regard to any interstitial lung process, one should always have a transbronchial biopsy before open lung biopsy.”
C. “No, open lung biopsy is a last resort in the workup of interstitial lung diseases.”
D. “No; given your chest x-ray and other factors, sarcoidosis is the likely diagnosis; yield from transbronchial biopsy should be around 90%.”

Key Concept/Objective: To understand the diagnostic approach to sarcoidosis

Given the demographics of this patient—a middle-aged African-American woman—sarcoidosis is a very likely diagnosis. This patient’s chest x-ray indicates that she has stage II sarcoidosis; patients with stage II sarcoidosis have a 90% chance of having their diagnosis confirmed by transbronchial biopsy. The biopsy will show noncaseating granulomas. In general, transbronchial biopsy is most useful in the diagnosis of sarcoidosis or diffuse infiltrative lung diseases of infectious cause. If the working diagnosis is neither infection nor sarcoidosis, then lung biopsy would likely be indicated. Open lung biopsy is a very invasive procedure and should be reserved for other types of diffuse infiltrative lung disease. (Answer: D—“No; given your chest x-ray and other factors, sarcoidosis is the likely diagnosis; yield from transbronchial biopsy should be around 90%.”)

26. A 38-year-old white man is referred to you for treatment of sarcoidosis. The patient reports that he has decreased exercise tolerance as well as a chronic cough. The patient has an 11-year history of injecting drug abuse. He brings records from his previous physician, which include a report of negative results on an HIV test, a chest x-ray report that reads, “diffuse interstitial process without hilar adenopathy,” and a pathology report of noncaseating granulomas that gave sarcoidosis as the final diagnosis.

Of the following, which is the most appropriate approach to the treatment of this patient?

A. Start prednisone, 60 mg q.d., and taper over 6 weeks
B. Inform the patient that given his chest x-ray findings, he has a 65%
chance of spontaneous remission
C. Assess arterial blood gases; if the patient is not hypoxic, schedule a follow-up appointment in 3 to 6 months
D. Obtain a biopsy specimen for further testing

Key Concept/Objective: To understand the differential diagnosis of illicit drug abuse and sarcoidosis

This patient’s demographics do not support the diagnosis of sarcoidosis; however, that diagnosis should not be ruled out. The pathologic changes in the lung seen with injecting drug abuse are secondary to talc, which is used as “filler,” most commonly with heroin. Given this patient’s history, the biopsy specimen should be examined under polarizing light; talc particles are characteristically seen with this technique. No treatment should be initiated until the proper diagnosis has been made. Patients with stage III sarcoidosis have a 33% chance of spontaneous resolution in 2 years. Although measurement of arterial blood gases is not contraindicated, this is clearly not the most appropriate step to take next in this patient’s workup. It is important to note that heroin per se is not associated with diffuse lung injury. (Answer: D—Obtain a biopsy specimen for further testing)

27. A 64-year-old white man presents to your office with a 1-year history of worsening dyspnea on exertion and mild cough, described as nonproductive. The patient has no other complaints. He reports he may have been exposed to asbestos when he was in his 20s, while working in a shipyard. The patient has never smoked. He has been treated with several inhaled beta agonists, without any improvement. The physical examination is significant for dry inspiratory crackles and clubbing of his digits. A chest x-ray shows a diffuse infiltrative process, without adenopathy or effusions. Ultimately, the patient undergoes an open lung biopsy, which shows minimal inflammatory round cell infiltrate, widening of alveolar septa, and fibrosis with fibroblastic foci.

What is the most likely diagnosis for this patient?
A. Bronchiolitis obliterans organizing pneumonia (BOOP)
B. Desquamative interstitial pneumonitis (DIP)
C. Idiopathic pulmonary fibrosis (IPF)
D. Acute interstitial fibrosis (Hamman-Rich syndrome)

Key Concept/Objective: To understand the diagnosis and prognosis of IPF

This patient’s history and presentation is classic of IPF—an insidious loss of pulmonary function and the absence of signs or symptoms of a systemic process. His biopsy specimen report describes usual interstitial pneumonitis (UIP), which is characteristic of IPF. Besides a chest x-ray, high-resolution CT of the chest and pulmonary function testing are useful noninvasive ways to evaluate patients; however, an open lung biopsy is ultimately needed for diagnosis in the majority of patients. Older patients may be spared the morbidity of open lung biopsy if they have a classic presentation and features suggestive of IPF on transbronchial biopsy. Survival is usually 2 to 3 years after diagnosis has been made. Although multiple medicinal therapies have been tried, none to date have improved survival in this patient population. Lung transplantation is a good option, and patients should be referred once the diffusing capacity of the lung for carbon monoxide (DLco) has dropped below 40%. Patients with DIP are usually younger; biopsy in these patients reveals a homogeneous pattern of involvement and characteristic pigmented alveolar macrophages. In patients with Hamman-Rich syndrome, cough and dyspnea rapidly progress to significant respiratory compromise. The onset of BOOP is more acute, and systemic symptoms such as fever and malaise are not uncommon; microscopic findings are distinct in BOOP. (Answer: C—Idiopathic pulmonary fibrosis [IPF])

28. A 34-year-old white man presents to the emergency department with a cough of abrupt onset, fever, and chest pain. He has no significant medical history. He is admitted to the intensive care unit, where his respiratory distress worsens to the point that he requires intubation. The patient’s chest radiograph shows diffuse, patchy ground-glass opacities and intralobular septal thickening. Bronchoscopy with bronchoalveolar lavage (BAL) shows copious amounts of grossly turbid exudates in the airways with material that tests positive with periodic acid–Schiff (PAS) reagent on pathologic examination.

What is the most likely diagnosis for this patient

A. Alveolar proteinosis
B. Löffler syndrome
C. Diffuse alveolar hemorrhage
D. Lymphangioleiomyomatosis

Key Concept/Objective: To understand the diagnosis of alveolar proteinosis

Clinical presentations of patients with alveolar proteinosis can vary greatly. The condition may progress, remain stable, or resolve spontaneously. Some patients are asymptomatic; others have severe respiratory insufficiency. Most patients present with gradually progressive exertional dyspnea and cough that is usually unproductive. Diagnosis is made with BAL, which shows grossly turbid exudates in the airways and PAS-positive material on pathologic examination. Löffler syndrome is characterized by transient and migratory infiltrates on chest x-ray and a predominance of eosinophils on BAL. Diffuse alveolar hemorrhage is associated with a history of hemoptosis or is evidenced by bleeding at the time of BAL. Lymphangioleiomyomatosis affects women only. (Answer: A— Alveolar proteinosis)

29. A 76-year-old Iranian woman with hypertension and chronic back pain presents with a nonproductive cough of 5 months’ duration and progressive dyspnea on exertion, which she has been experiencing for
2 months. She denies other symptoms, including weight loss, fever, and night sweats. Her medications include gabapentin, triamterene-hydrochlorothiazide, atenolol, and premarin. Her examination is normal. An x-ray done in clinic reveals diffuse interstitial infiltrates without adenopathy or effusion.

Which of the following is the most likely diagnosis for this patient?
A. Drug-induced infiltrative lung disease
B. Tuberculosis
C. Sarcoidosis
D. Idiopathic pulmonary fibrosis
E. Pulmonary lymphangitic carcinomatosis

Key Concept/Objective: To be able to differentiate the natural history, associated symptoms, and x-ray findings of common causes of infiltrative lung diseases

Given the absence of a causative medication, systemic symptoms, hilar adenopathy, or pleural effusion, the gradually progressive cough and dyspnea in this older patient is most likely being caused by idiopathic pulmonary fibrosis. Lymphagitic carcinomatosis and tuberculosis are typically subacute; these disorders are associated with systemic symptoms and can present as pleural effusions. Because idiopathic pulmonary fibrosis is a diagnosis of exclusion, screening for breast and colon cancer and administering a PPD test are appropriate steps in this patient’s evaluation. Although sarcoidosis does not typically present in the elderly, it can do so. Stage III sarcoidosis (diffuse infiltrative lung disease without adenopathy) is a less common form of sarcoidosis than the other stages and is often associated with extrapulmonary manifestations. (Answer: D— Idiopathic pulmonary fibrosis)

30. A 28-year-old Asian man presents with a nonproductive cough of 3 weeks’ duration and progressive dyspnea on exertion. His dyspnea became acutely worse today. He denies having other symptoms, including fever and weight loss. He used intravenous cocaine on three occasions. He works on the docks unloading cargo ships. He is on no medications but smokes one pack of cigarettes a day. Chest x-ray shows bilateral interstitial infiltrates and a small left pneumothorax.

Which of the following would be initial therapy for this patient?
A. Erythromycin
B. Prednisone
C. Dapsone
D. Trimethoprim-sulfamethoxazole
E. Smoking cessation

Key Concept/Objective: To know that Pneumocystis carinii pneumonia (PCP) is a common cause of subacute diffuse interstitial lung disease, especially in the setting of pneumothorax

This patient most likely has PCP, given the subacute clinical course, the pneumothorax, and this patient’s risk factors for HIV. Even had the patient not been forthcoming with his risk factors, PCP would still be the presumptive diagnosis until another cause was identified. A pneumothorax in the setting of diffuse infiltrative lung disease is a clue to eosinophilic granuloma, PCP, and lymphangiomatosis. Eosinophilic granuloma (pulmonary histiocytosis X) occurs in patients between the ages of 20 and 40 years and is associated with smoking. (Answer: D—Trimethoprim-sulfamethoxazole)

31. A 23-year-old black woman fell while snowboarding and hurt her ribs. She denies having had any symptoms before her fall. Her examination reveals chest wall tenderness but no adenopathy, skin lesions, or organomegaly. An x-ray done to look for a rib fracture reveals bilateral hilar adenopathy.

Which of the following would be appropriate as initial therapy for this patient?

A. Isoniazid
B. Itraconazole
C. Prednisone
D. Chloroquine
E. Observation

Key Concept/Objective: To know the presentation and differential diagnosis of stage I sarcoidosis

This asymptomatic young woman most likely has sarcoidosis. The differential diagnosis includes lymphoma, tuberculosis, histoplasmosis, and other granulomatous infections. In the absence of systemic symptoms or examination findings, these other diseases are unlikely. Skin tests for both tuberculosis and histoplasmosis should be performed; if the results are negative, no further workup would be needed at this point. In the absence of symptoms or hypercalcemia, this patient can be observed clinically without initial intervention. She has a 75% chance of clinical remission over the next 2 years. (Answer: E—Observation)

32. A 45-year-old dairy farmer reports that he experiences coughing and wheezing immediately upon entering the barn each morning. These symptoms resolve after returning home each evening.

Which of the following is the most likely diagnosis for this patient?

A. Acute hypersensitivity pneumonitis
B. Acute interstitial pneumonia
C. Allergen-induced bronchospasm
D. Lymphocytic interstitial pneumonia
E. Eosinophilic granuloma

Key Concept/Objective: To understand the course of hypersensitivity pneumonitis

The differential diagnosis for symptoms that come and go in conjunction with exposure to an unidentified causative agent includes hypersensitivity pneumonitis and irritantor allergen-induced lung irritation. Hypersensitivity pneumonitis is mediated by the complement pathway and takes 4 to 8 hours to develop after antigenic exposure. Antigen-induced bronchospasm is mediated by histamines and occurs soon after exposure. (Answer: C—Allergen-induced bronchospasm)

For more information, see Staton GW, Ingram RH Jr: 14 Respiratory Medicine: V Chronic Diffuse Infiltrative Lung Disease. ACP Medicine Online (www.acpmedicine.com). Dale DC, Federman DD, Eds. WebMD Inc., New York, November 2004

Ventilatory Control during Wakefulness and Sleep

33. A 64-year-old man with known severe chronic obstructive pulmonary disease is admitted to the hospital with worsening shortness of breath and green sputum production. He is known to require home oxygen therapy. He is given nebulized bronchodilators, antibiotics, intravenous methylprednisolone, and high-flow oxygen therapy. Although his bronchospasm is resolving and his respiration is less labored, you notice a decreased sensorium. Arterial blood gas measurements confirm the presence of hypercapnia and hypoxia.

Which of the following statements regarding the physiology of ventilatory control is false?
A. The central neural network is composed of medullary neurons, pontine neurons, and the nucleus tractus solitarius (NTS)
B. Carotid and aortic bodies are peripheral chemoreceptors that are stimulated primarily by hypocapnia
C. Cerebrospinal fluid can decrease its pH in response to increased levels of arterial carbon dioxide tension (PaCO2) to stimulate cells on the ventral medullary surface
D. Mechanoreceptors in the upper airway, chest wall, and lung detect mechanical deformation and temperature changes that result from inhalation and exhalation

Key Concept/Objective: To understand the key concepts of the physiology of ventilatory control

Ventilation is a critical function for eliminating carbon dioxide and acquiring oxygen. The control system for ventilation not only optimizes gas exchange but also serves a role in acid-base homeostasis, speech, deglutition, defecation, and postural adjustments. Inhalation and exhalation begin with the discharge properties of putative pacemaker neuronal groups located in the medulla. This neural network is embedded in a system of adjacent medullary neurons, pontine neurons, and regions such as the NTS that receive neural impulses through lung inflation, lung deflation, blood pressure, and other afferent systems. The intensity of the activity of medullary neurons is affected by chemoreceptors. The peripheral chemoreceptors—the carotid and aortic bodies—are highly vascular collections of specialized sensory cells. The carotid bodies are located bilaterally at the bifurcations of the common carotid arteries; the aortic bodies are situated anterior and posterior to the arch of the aorta and left main pulmonary artery. The peripheral chemoreceptors are stimulated primarily by a low arterial oxygen tension (PaO2), although hypercapnia, acidemia, and, possibly, hyperthermia may influence the gain of the response to hypoxemia. Impulses travel from the carotid and aortic bodies to the NTS via sensory ganglia and the afferent nerves that follow along the ninth and 10th cranial nerves, respectively. Increases in PaCO2 stimulate cells on the ventral medullary surface, primarily by lowering the pH of the medullary extracellular fluid. Specialized sensory cells (mechanoreceptors) located in the upper airway, chest wall, and lung detect mechanical deformation and temperature changes resulting from inhalation and exhalation. (Answer: B—Carotid and aortic bodies are peripheral chemoreceptors that are stimulated primarily by hypocapnia)

34. A 43-year-old man comes to your office for evaluation of a dry cough and worsening shortness of breath of 3 months’ duration. He denies having orthopnea, paroxysmal nocturnal dyspnea, or lower extremity edema. He has no history of smoking or cardiac disease. On examination, the patient is tachypneic, but all other vital signs are normal. There is no clubbing, and the cardiac examination is normal, but you notice Velcro-like crackles at both lung bases. Arterial blood gas measurements reveal hypoxemia, hypocarbia, and a respiratory alkalosis. High-resolution CT scanning of the chest reveals diffuse interstitial infiltrates.

Which of the following statements regarding ventilatory drive is false?
A. Interstitial lung diseases can be accompanied by hyperventilation that results from a rapid, shallow breathing pattern
B. Adaptation to chronic hypoventilation in sleep apnea, chronic obstructive pulmonary disease (COPD), neuromuscular disease, and chest wall disease my depress responsiveness to CO2
C. Metabolic causes of hypoventilation may include metabolic alkalosis, deficiency of thyroid hormone, and excess sedative or narcotic agents
D. Hyperventilation caused by progesterone stimulation is the result of an increase in both tidal volume and respiratory rate

Key Concept/Objective: To understand ventilatory drive

Interstitial lung diseases (e.g., pulmonary fibrosis) increase resting ventilation and lower PaCO2 as a result of increased activity of lung receptors (probably C fibers). The hyperventilation that accompanies pulmonary edema, pneumonia, interstitial disease, and the acute respiratory distress syndrome is a rapid, shallow breathing pattern that results from activation of these lung receptors. Hyperventilation is regularly produced by exposure to high altitude or other hypoxic environments; metabolic acidosis; pregnancy and conditions associated with elevated progestational hormones; anxiety states; and mildly toxic doses of salicylates, amphetamines, or other drugs that stimulate the central nervous system. Unlike the hyperventilation associated with parenchymal lung disease, the hyperventilation that occurs during progesterone stimulation (such as occurs during pregnancy) and metabolic acidosis is associated with an increase in tidal volume and little increase in respiratory rate. Hypoventilation occurs when alveolar ventilation is insufficient to eliminate metabolically produced CO2. Metabolic causes of hypoventilation may include metabolic alkalosis, deficiency of thyroid hormone, and excess doses of sedative and narcotic agents. As hypoventilation becomes chronic, adaptation of receptors, of central inspiratory neurons, of metabolic alkalosis, or of all three may occur. Adaptation to chronic hypoventilation in sleep apnea, COPD, neuromuscular disease, and chest wall disease may eventually reduce responsiveness to CO2 and depress ventilation during rest. (Answer: D—Hyperventilation caused by progesterone stimulation is the result of an increase in both tidal volume and respiratory rate)

35. A 55-year-old man presents to your clinic for evaluation of chronic headache and daytime sleepiness. His wife reports a long history of snoring, and the patient is concerned that he might have sleep apnea because he is overweight and his brother was recently diagnosed with sleep apnea.

Which of the following statements regarding sleep apnea is false?
A. There are three categories of sleep apnea: central, obstructive, and mixed
B. Patients with sleep apnea are not at increased risk for developing hypertension
C. Sleep apnea has a genetic component
D. Patients who have a short mandible and a round head are predisposed to apnea

Key Concept/Objective: To understand risk factors for and potential complications of sleep apnea

Three patterns of apnea, or cessation of breathing, can be observed during sleep. These apneas are defined as episodes of a reduction in airflow of more than 80% that occur for more than 10 seconds. Apneas may be classified as central (or nonobstructive), obstructive, or mixed. Certain measures are used to quantify respiratory disturbances during sleep. The apnea-hypopnea index (AHI) is the total number of apneas and hypopneas that occur during sleep, divided by the hours of sleep time. In communities in the United States, 9% to 12% of women and 27% to 35% of men may have an AHI higher than 5—a number often quoted as a threshold value for normality; however, many people with an AHI higher than 5 have no symptoms or apparent illness. If the definition of illness is the presence of daytime sleepiness or cardiovascular complications such as hypertension, the estimates are that approximately 2% of women and 4% of men have symptomatic sleep-disordered breathing (SDB). Snoring is generally considered a predisposing feature for the development of SDB and sleep apnea. Sleep apnea has a genetic component. Symptoms relating to apnea occur two to four times more often in family members of affected patients than in a control population. Individuals with craniofacial features of a short mandible and round head are predisposed to snoring, apneas, or both. There are familial traits in hypercapnic and hypoxic sensitivity; these could relate to the tendency to breathe periodically during sleep. Sleep apnea is more common in stroke patients. (Answer: B—Patients with sleep apnea are not at increased risk for developing hypertension)

For more information, see Strohl KP: 14 Respiratory Medicine: VI Ventilatory Control during Wakefulness and Sleep. ACP Medicine Online (www.acpmedicine.com). Dale DC, Federman DD, Eds. WebMD Inc., New York, December 2003

Disorders of the Chest Wall

36. A 43-year-old white man with a history of diabetes and hypertension presents to your office for a routine follow-up visit. He complains of increasing dyspnea on exertion but denies having cough or edema. The patient has a long history of morbid obesity. His body mass index is 35 kg/m2. You initiate a workup for his dyspnea on exertion that includes pulmonary function tests, arterial blood gas measurements, an echocardiogram, and assessment of his hematocrit and serum chemistries.

Which of the following statements regarding obesity and its impact on respiratory function is true?

A. In the absence of other primary lung illness, the major impact of obesity on respiration is as an obstructive respiratory defect
B. Obesity typically causes an increase in functional residual capacity
(FRC)
C. The key therapy for patients with respiratory problems related to obesity is to decrease their physical activity to a level that is comfortable for the patient
D. Obese patients may experience significant dyspnea during exercise because of the increased work required to move the heavy chest and abdomen and because of overall poor conditioning

Key Concept/Objective: To know the implications of obesity on respiratory function

Obesity has several marked effects on respiratory function and can be a cause of severe respiratory disease. Obesity imposes a restrictive load on the thoracic cage, both directly, because weight has been added to the rib cage, and indirectly, because the large abdominal panniculus impedes the motion of the diaphragm when the person is supine. In the absence of other primary lung illnesses, obstructive respiratory disease is not a prominent feature of obesity-related respiratory disease. Obesity characteristically causes a decrease in FRC because of the increased load applied to the chest wall. Weight loss is the most important therapy for patients with respiratory problems related to obesity. Conservative measures for weight loss, such as improving diet and moderate exercise, should be the initial approach. Decreasing physical activity would likely induce further weight gain and worsening of aerobic conditioning. Obese patients may experience significant dyspnea during exercise, because of the increased work required to move the heavy chest and abdomen and because of overall poor conditioning. (Answer: D—Obese patients may experience significant dyspnea during exercise because of the increased work required to move the heavy chest and abdomen and because of overall poor conditioning)

37. A 21-year-old white woman with kyphoscoliosis visits your office to establish primary care. She recently moved to the area and states that she has been relatively healthy and was provided appropriate vaccinations and screenings by her previous physician. She developed scoliosis during her early teenage years. She denies having knowledge of any previous complications from her condition. She asks you to explain her condition and its possible complications.

Which of the following statements regarding kyphoscoliosis is true?
A. The two distinct forms of costovertebral skeletal abnormalities— scoliosis and kyphosis—do not typically occur together in a given patient
B. Approximately 80% of cases of kyphoscoliosis are idiopathic
C. Idiopathic kyphoscoliosis is most commonly a congenital abnormality or an abnormality that develops in the aged population
D. The incidence of kyphoscoliosis is distributed equally between the sexes

Key Concept/Objective: To know the features of idiopathic kyphoscoliosis

Kyphoscoliosis is an illness that can be associated with mild to severe respiratory compromise. The two basic types of costovertebral skeletal deformity—scoliosis, a lateral curvature with rotation of the vertebral column, and kyphosis, an anterior flexion of the spine—are usually found in combination. Approximately 80% of cases of kyphoscoliosis are idiopathic. Idiopathic kyphoscoliosis commonly begins in late childhood or early adolescence and may progress in severity during these years of rapid skeletal growth. Idiopathic kyphoscoliosis is not to be confused with kyphoscoliosis caused by a known underlying condition, such as osteoporosis or compression fractures in elderly patients. The incidence of kyphoscoliosis in females is four times higher than that in males. (Answer: B—Approximately 80% of cases of kyphoscoliosis are idiopathic)

38. A 37-year-old man arrives at your emergency center by ambulance shortly after being involved in a motor vehicle accident. The emergency medical technician (EMT) reports that the patient is hemodynamically stable with minimal external blood loss and no loss of consciousness. The EMT reports that the patient appears to be in moderate to severe respiratory distress; the patient has a respiratory rate of
40 breaths/min and an O2 saturation of 78% while receiving supplemental oxygen at a rate of 3 L/min by nasal cannula. On physical examination, you note a remarkable 15 cm right anterolateral chest contusion. The contused segment appears to move paradoxically with respect to respiration. The patient has clear bilateral breath sounds in the upper and lower regions of both lungs.

Which of the following statements regarding flail chest injury is most accurate for this patient?
A. In young, otherwise healthy patients, a large flail chest segment is not a life-threatening injury
B. The most appropriate step to take next in treatment of this patient
is to provide supplemental oxygen by 100% nonrebreathing mask to attain O2 saturations greater than 90%
C. The most appropriate step to take next in the treatment of this patient is to provide positive pressure ventilation
D. The most appropriate step to take next in the treatment of this patient is to order and evaluate a stat portable chest x-ray to rule out a tension pneumothorax

Key Concept/Objective: To understand emergent therapy of flail chest segment with respiratory failure

Flail chest is an acute process that may lead to life-threatening abnormalities of gas exchange and mechanical function. This patient is in acute respiratory failure as a result of the massive chest-wall trauma and resultant flail segment. Stability of the thoracic cage is necessary for the muscles of inspiration to inflate the lung. In flail chest, a locally compliant portion of the chest wall moves inward as the remainder of the thoracic cage expands during inhalation; the same portion then moves outward during exhalation. Consequently, tidal volume is diminished because the region of lung associated with the chest wall abnormality paradoxically increases its volume during exhalation and deflates during inhalation. The result is progressive hypoxemia and hypercapnia. Multiple rib fractures, particularly when they occur in a parallel vertical orientation, can produce a flail chest. The degree of dysfunction is directly proportional to the volume of lung involved in paradoxical motion. Patient management may be complicated by other manifestations of trauma to the chest, such as splinting of ventilation because of pain, contusion of the underlying lung, or hemothorax or pneumothorax. Positive-pressure inflation of the lung or negative pressure applied to the chest wall corrects the abnormality until more definitive stabilization procedures can be undertaken. Supplemental O2 by face mask will not alleviate the patient’s paradoxical chest movement and loss of tidal volume. For this patient, the most appropriate step to take next is to provide positive-pressure ventilation, preferably with endotracheal intubation. A chest x-ray is needed in this patient, but ventilatory resuscitation should take precedence. The fact that the patient has good bilateral breath sounds and is hemodynamically stable would make a significant tension pneumothorax unlikely. (Answer: C—The most appropriate step to take next in the treatment of this patient is to provide positive pressure ventilation)

39. A 51-year-old man presents to your office for evaluation of a nonproductive cough and a “scratchy” throat. He has no significant medical history but does have a 50-pack-year history of cigarette smoking. He states that his cough and mild sore throat started 3 weeks ago when he developed a “head cold” and that he has had a persistently runny nose since. After further questioning, you come to the conclusion that it is likely that this patient’s cough is secondary to postnasal drip following his upper respiratory infection. Because of his significant smoking history, you order a routine chest x-ray, which is interpreted as being normal except for an elevated right hemidiaphragm. You suspect a diaphragmatic paralysis, which is verified by a sniff test.

Which of the following statements regarding diaphragmatic paralysis is true?
A. With bilateral diaphragmatic paralysis, degrees of dyspnea tend to be unrelated to body position
B. In otherwise healthy patients, unilateral diaphragmatic paralysis will typically present as acute onset of orthopnea and dyspnea on exertion
C. The underlying cause of bilateral diaphragmatic paralysis is almost exclusively trauma related
D. Most cases of unilateral diaphragmatic paralysis are the result of neoplastic invasion of the phrenic nerve

Key Concept/Objective: To know the characteristics of unilateral and bilateral diaphragmatic paralyses

Diaphragmatic paralysis is a relatively common finding. Most cases of unilateral diaphragmatic paralysis are the result of neoplastic invasion of the phrenic nerve. Compression or destruction of the phrenic nerve by surgery, trauma, or enlargement of lymph nodes or aneurysmal vessels may also cause the condition. Bilateral diaphragmatic paralysis can result from a number of causes, including cervical and thoracic surgery, cold cardioplegia for cardiac surgery, trauma, multiple sclerosis, and neuralgic amyotrophy. Orthopnea may be a prominent symptom in the setting of bilateral diaphragm dysfunction. With the patient supine, the hydrostatic force of the abdominal contents pushes the patient’s diaphragm into the thorax. Negative intrapleural pressure generated by the accessory muscles causes the diaphragm to be sucked further into the thorax during inspiration, producing a paradoxical inward motion of the upper abdomen as the thorax expands. As a result, mechanical and gas exchange abnormalities similar to those seen in flail chest develop. Unilateral diaphragmatic paralysis is most often detected as an asymptomatic radiographic finding. In the absence of associated pleuropulmonary disease, most adult patients with unilateral diaphragmatic paralysis but without a coexisting pulmonary disease remain asymptomatic. (Answer: D—Most cases of unilateral diaphragmatic paralysis are the result of neoplastic invasion of the phrenic nerve)

40. An 82-year-old woman who resides in a nursing home presents to your office for routine evaluation. She has severe kyphoscoliosis secondary to osteoporosis, and she has been hospitalized twice for pneumonia.

Which of the following is NOT important in the prevention of respiratory compromise in this patient?
A. Immunization with influenza vaccine
B. Early treatment of respiratory tract infections
C. Avoidance of central nervous system depressants
D. Nocturnal oxygen therapy
E. Yearly pulmonary function testing to monitor progress

Key Concept/Objective: To understand outpatient management of kyphoscoliosis-induced respiratory disorders

Kyphoscoliosis is the most common disorder of the chest wall that produces ventilatory failure. Approximately 80% of cases are idiopathic, first manifesting in late childhood and early adolescence; the other 20% are caused by neuromuscular disorders. Females are four times more likely to develop this deformity than males. The deformities worsen with age. Immunization with influenza and pneumococcal vaccines, early treatment of respiratory infections, avoidance of CNS depressants, and use of nocturnal oxygen therapy can prolong life and enhance quality of life in these patients. There is no evidence that yearly pulmonary function testing will affect the disease process or provide any added information for intervention. (Answer: E—Yearly pulmonary function testing to monitor progress)

41. A 36-year-old black man presents to the emergency department after a motor vehicle accident. Except for some minor scrapes, he is asymptomatic; however, his chest x-ray reveals an elevated left hemidiaphragm.

Which of the following tests can confirm the diagnosis?
A. Determination of DLco on pulmonary function testing
B. Arterial blood gas measurements
C. Sniff test
D. Pulse oximetry
E. CT scan of the chest

Key Concept/Objective: To understand the evaluation and diagnosis of a patient with unilateral diaphragmatic paralysis

Unilateral diaphragmatic paralysis is most often detected as a radiographic finding in an asymptomatic patient. Although most cases are the result of neoplastic invasion of the phrenic nerve, it is also commonly seen in postoperative patients, in patients with trauma, or in idiopathic cases. The sniff test involves asking the patient to perform a sudden, forceful inspiration. The diaphragmatic movements can be viewed under fluoroscopy; with unilateral diaphragmatic paralysis, the affected side of the diaphragm ascends into the thorax. This movement is in the opposite direction of the normal side. Such a finding confirms the diagnosis of diaphragmatic paralysis. (Answer: C—Sniff test)

42. A 26-year-old woman presents to the emergency department with shortness of breath, which has been progressively increasing for several days. She has also been experiencing increasing weakness and double vision. She notes a worsening of her symptoms at the end of the day, and she has noticed weakness while brushing her hair. Her physical examination is unrevealing.

Which of the following neuromuscular disorders is most likely the cause of this patient’s symptoms

A. Guillain-Barré syndrome
B. Bilateral diaphragmatic paralysis
C. Myasthenia gravis
D. Duchenne muscular dystrophy
E. Amyotrophic lateral sclerosis (ALS)

Key Concept/Objective: To be able to differentiate between the multiple neuromuscular disorders that affect respiratory function

Guillain-Barré syndrome usually presents as an ascending paralysis. Although bilateral diaphragmatic paralysis would explain this patient’s shortness of breath, the proximal muscle weakness and ocular symptoms would remain unexplained. Duchenne muscular dystrophy is an X-linked disorder that exclusively affects males. Its symptoms are present by 3 to 5 years of age, and patients are usually wheelchair bound by 12 years of age. The majority of patients with ALS present clinically with progressive asymmetrical weakness, fasciculations, and prominent muscle atrophy. The distal musculature is primarily involved. Myasthenia gravis is an autoimmune disorder that affects the neuromuscular junction: specifically, the postsynaptic acetylcholine receptor. Patients usually present with intermittent symptoms that are usually worse at the end of the day. Respiratory failure may occur; in myasthenia crisis, the patient requires a ventilator. (Answer: C—Myasthenia gravis)

For more information, see Staton GW, Ingram RH Jr: 14 Respiratory Medicine: VII Disorders of the Chest Wall. ACP Medicine Online (www.acpmedicine.com). Dale DC, Federman DD, Eds. WebMD Inc., New York, December 2003

Respiratory Failure

43. A 41-year-old woman presents to the emergency department for evaluation of shortness of breath. She is currently undergoing therapy for newly diagnosed breast cancer. She states that she was in her usual state of health until she began to experience acute shortness of breath 2 hours ago. For the past 2 hours, she has also been experiencing sharp right chest pain on inspiration. She denies having fever, chills, or cough. Results of physical examination are as follows: heart rate, 130 beats/min; respiratory rate, 30 breaths/min; a loud second heart sound; and there is mild pretibial pitting edema of the left lower extremity. Results of blood gas measurements are as follows: normal pH; arterial carbon dioxide tension (PaCO2), 17 mm Hg; arterial oxygen tension (PaO2), 70 mm Hg; and hemoglobin O2 saturation, 95%. The patient is started on anticoagulation therapy with heparin, and a helical CT scan of the chest is ordered.

Which of the following statements regarding acute hypoxemic respiratory failure is true?

A. This patient has no significant V/Q mismatching because her hemoglobin saturation is normal
B. In patients with ARDS, shunting is the major physiologic derangement resulting in hypoxemia
C. Pure alveolar hypoventilation is the most common pathophysiologic cause of acute hypoxemia
D. Shunting and V/Q mismatching respond similarly to inhalation of 100% O2

Key Concept/Objective: To know the clinical characteristics of common causes of hypoxemia

Patients with ARDS can have diffusion impairments that contribute to hypoxemia, but shunting is the more important physiologic derangement in this disorder. The alveolararterial oxygen gradient or difference (A-aDO2) is used to identify V/Q mismatching when the measured PaO2 is normalized by hyperventilation. V/Q mismatching is the most common pathophysiologic cause of acute hypoxemia. It develops when there is a decrease in ventilation to normally perfused regions of the lung, a decrease in perfusion to normally ventilated regions of the lung, or some combination of a decrease in both ventilation and perfusion. Shunting can be differentiated from V/Q mismatching on the basis of the differences in the response to inhalation of 100% oxygen. (Answer: B—In patients with ARDS, shunting is the major physiologic derangement resulting in hypoxemia)

44. A 74-year-old male patient of yours who has severe COPD presents to your office for the evaluation of worsening shortness of breath. The patient has smoked two packs of cigarettes daily for the past 50 years. Through home oxygen therapy, he receives oxygen at a rate of 2 L/min. He states that he was in his usual state of health until 2 days ago, when he developed worsening shortness of breath, particularly with exertion. He also complains of mild substernal “burning” pain with exertion. He denies having orthopnea, edema, or palpitations. His hemoglobin O2 saturations are 92% on 2 L of oxygen provided by nasal cannula. Results of blood gas measurements are as follows: pH, 7.38; PaCO2, 80 mm Hg; and PaO2, 70 mm Hg. ECG shows lateral T wave inversions; otherwise, ECG results are unremarkable.

For this patient, which of the following statements regarding hypercapnic respiratory failure is true?
A. This patient should be admitted to the hospital because he has acute hypercapnic respiratory failure and will likely require mechanical ventilatory support
B. As with acute hypoxemia, the effects of hypercapnia on the central nervous system are typically irreversible
C. Acute hypercapnic respiratory failure is defined as a PaCO2 greater than 45 to 50 mm Hg along with respiratory acidosis
D. Acute elevation in PaCO2 to 80 or 90 mm Hg is generally well tolerated, but levels in excess of 100 mm Hg often produce neurologic signs and symptoms

Key Concept/Objective: To understand the clinical effects and the management of acute and chronic hypercapnia

Acute hypercapnic respiratory failure is defined as a PaCO2 greater than 45 to 50 mm Hg along with respiratory acidosis. Signs and symptoms of hypercapnia depend not only on the absolute level of PaCO2 but also on the rate at which the level increases. A PaCO2 above 100 mm Hg may be well tolerated if the hypercapnia develops slowly and acidemia is minimized by renal compensatory changes, as is the case with this patient. Acute elevation in PaCO2 to 80 to 90 mm Hg may produce many neurologic signs and symptoms, including confusion, headaches, seizures, and coma. A careful neurologic examination of a patient with acute hypercapnia may reveal agitation, coarse tremor, slurred speech, asterixis, and, occasionally, papilledema. These effects of hypercapnia on the central nervous system are fully reversible, unlike the potentially permanent neurologic sequelae that are associated with acute hypoxemia. (Answer: C—Acute hypercapnic respiratory failure is defined as a PaCO2 greater than 45 to 50 mm Hg along with respiratory acidosis)

45. A 52-year-old man with severe emphysema presents to the emergency department with shortness of breath and altered mental status. A history is taken from the patient’s wife. She states that the patient was in his usual state of health until 24 hours ago, when he awoke with fever and shortness of breath. Since that time, he has experienced worsening fever, cough, and sputum production. She states that the patient has been acting “very funny” for the past several hours. She does not believe the patient has come into contact with anyone who was sick, and she states that he receives oxygen at home at a rate of 3 L/min via nasal cannula. On physical examination, the patient’s temperature is found to be 101.1° F (38.4° C). The oropharynx and mucous membranes are dry, and rales with egophony are heard at the left pulmonary base. Laboratory studies reveal leukocytosis with left shift. Results of arterial blood gas measurements are as follows: pH, 7.02; PaCO2, 80 mm Hg; and PaO2, 60 mm Hg.

Which of the following statements regarding the management of respiratory failure in patients with COPD is true?
A. The most common cause of acute respiratory deterioration in patients with COPD is cigarette smoking
B. The first priority in the management of respiratory failure in these patients is to decrease PaCO2 to a normal value
C. The level of PaCO2 at which ventilatory assistance becomes necessary is approximately 70 mm Hg for males and 60 mm Hg for females
D. When invasive ventilation is required, PaCO2 levels should not be lowered to the normal range in patients with chronic hypercapnia

Key Concept: To understand the management of respiratory failure in patients with COPD

The first priority is to achieve a PaO2 level of 50 to 60 mm Hg but no higher. Intubation should be performed if hemodynamic instability or somnolence occurs or if secretions cannot be cleared. It is important to remember that PaCO2 levels in patients with chronic hypercapnia should not be lowered to the normal range, because this could result in alkalemia, which increases the risk of cardiac dysrhythmias and seizures. In addition, overventilation for more than 2 to 3 days may result in renal restoration of the pH to normal. As a consequence, during subsequent trials of spontaneous ventilation, as the PaCO2 rises to the baseline hypercapnic level, the patient becomes acidemic or the patient’s respiratory muscles become fatigued because of the greater minute ventilation required for the reset baseline pH and PaCO2. (Answer: D—When invasive ventilation is required, PaCO2 levels should not be lowered to the normal range in patients with chronic hypercapnia)

46. A 29-year-old man with AIDS is admitted to the hospital for worsening shortness of breath. He was in his usual state of health until 1 week ago, when he developed dyspnea on exertion, with cough productive of thick sputum. His dyspnea has worsened over the past week, and he has developed a fever as well. He denies having been in contact with sick persons. He states that he received treatment for “PCP” 1 year ago. He also states that his last CD4+ T cell count was “less than 10.” He is not currently taking any medications, because he cannot afford them. On his second day of hospitalization, he develops acute respiratory failure and is moved to the critical care unit, where he is intubated and undergoes mechanical ventilation.

Which of the following statements regarding complications of mechanical ventilation is true?

A. Ventilated patients with acute, worsening respiratory distress or oxygen desaturation should be disconnected from the ventilator; manual ventilation should be administered with an anesthesia bag and 100% oxygen
B. Growth of cultures obtained by suctioning through an endotracheal tube leads to a reliable diagnosis of pneumonia in ventilated patients
C. Subcutaneous emphysema is the most common life-threatening manifestation of barotrauma
D. In a patient with whole-lung atelectasis, a chest radiograph will reveal increased opacity in the affected hemithorax, together with a contralateral tracheal shift

Key Concept/Objective: To know the complications of mechanical ventilation

Worsening respiratory distress or arterial oxygen desaturation may develop suddenly as a result of changes in the patient’s cardiopulmonary status or secondary to a mechanical malfunction. The first priority is to ensure patency and correct positioning of the patient’s airway so that adequate oxygenation and ventilation can be administered during the ensuing evaluation. The patient should be disconnected from the ventilator, and manual ventilation should be administered with an anesthesia bag, using 100% oxygen. Tension pneumothorax is the most common life-threatening manifestation of barotrauma. Tension pneumothorax leads to worsening hypoxemia and decreased venous return with hypotension. With atelectasis of an entire lung, breath sounds are diminished or absent on the affected side, and the trachea is shifted toward that side. (Answer: A— Ventilated patients with acute, worsening respiratory distress or oxygen desaturation should be disconnected from the ventilator; manual ventilation should be administered with an anesthesia bag and 100% oxygen)

For more information, see Kollef MH: 14 Respiratory Medicine: VIII Respiratory Failure. ACP Medicine Online (www.acpmedicine.com). Dale DC, Federman DD, Eds. WebMD Inc., New York, September 2004

Disorders of the Pleura, Hila, and Mediastinum

47. A 43-year-old male nurse presents to your office for evaluation. For the past 2 months, he has experienced intermittent fever, night sweats, and a 20-lb weight loss. He denies having any cough or sputum production. The patient states that about 3 months ago, he tested positive on purified protein derivative (PPD) screening. He denies any drug abuse, nor does he report any HIV risk factors. The patient states that he was prescribed isoniazid, but he chose not to follow this regimen. His chest x-ray is remarkable only for a moderate left pleural effusion.

Which of the following statements regarding tuberculous pleuritis is true?
A. Pleural effusion is more often a manifestation of reactivation tuberculosis than of primary tuberculosis
B. Without therapy, this patient’s pleural effusion will likely persist for many years
C. In most cases of this illness, pleural fluid cell differential will reveal greater than 85% neutrophils
D. Acid-fast bacilli are rarely seen in pleural liquid, and cultures are positive in only 20% to 40% of patients

Key Concept/Objectives: To understand the clinical features of tuberculous pleuritis and pleural effusion

Pleural effusion is more often a manifestation of primary tuberculosis than of reactivation tuberculosis. In patients with primary tuberculosis, untreated pleural effusions resolve spontaneously in approximately 2 to 4 months. However, active tuberculosis develops in two thirds of such patients during the ensuing 5 years. The pleural liquid is usually serous or serosanguineous. In most cases, the differential white cell count reveals lymphocytosis. Acid-fast bacilli are rarely seen in pleural liquid, and cultures are positive in only 20% to 40% of patients. However, closed-needle biopsy of the pleura reveals caseating or noncaseating granulomas in approximately 70% of cases and provides material that is culture positive in approximately 75% of cases. Thus, the total diagnostic yield, as determined on the basis of histopathology and culture, is 90% to 95%. (Answer: D—Acid-fast bacilli are rarely seen in pleural liquid, and cultures are positive in only 20% to 40% of patients)

48. A 21-year-old white man is admitted to the hospital. The patient reports that while jogging earlier that day, he developed acute right anterior chest pain that was significantly worsened by deep inspiration. His pain radiated to his left scapula. He also developed moderate shortness of breath. He denies having any fever or chills; he has not experienced any recent immobility, and he has no personal or family history of clotting disorders. The patient has smoked one pack of cigarettes a day for the past 3 years. A chest x-ray is normal except for a large left pneumothorax.

Which of the following statements regarding idiopathic spontaneous pneumothorax is true?
A. The peak incidence occurs in persons between 30 and 50 years of age; there is a strong female preponderance
B. Patients are often tall and thin in stature and are often cigarette smokers
C. Most patients with idiopathic spontaneous pneumothorax have subpleural basilar blebs
D. Strenuous physical activity and airplane travel are frequent triggers for the development of idiopathic spontaneous pneumothorax

Key Concept/Objectives: To understand the clinical features of idiopathic spontaneous pneumothorax

The peak incidence of idiopathic spontaneous pneumothorax is in persons between 20 and 30 years of age; the male-to-female ratio is approximately 5:1. Patients often have a tall, thin stature and very frequently are cigarette smokers. Although patients with idiopathic spontaneous pneumothorax are otherwise healthy, most have subpleural apical blebs, frequently associated with more diffuse centrilobular emphysema that is detectable by CT scan. Common misconceptions are that strenuous physical activity is frequently a trigger for the development of pneumothorax and that patients are at increased risk during airplane travel. In fact, most studies have found that the onset of symptoms of pneumothorax usually occurs at rest or during light activity. (Answer: B— Patients are often tall and thin in stature and are often cigarette smokers)

49. A 55-year-old man visits your office with a complaint of fatigue and increasing dyspnea on exertion. He has been experiencing these symptoms for 2 weeks. He denies having fever, chills, cough, or weight loss, and he has no significant cardiac history. He denies having been in contact with anyone who was ill. He recently quit smoking, after having smoked cigarettes for 35 years. He does have a history of alcoholism and chronic pancreatitis; the pancreatitis has been well controlled with analgesics and pancreatic enzyme replacement therapy. His serum chemistries and complete blood count are unremarkable. A chest x-ray reveals a large left pleural effusion. A diagnostic thoracentesis is performed.

Which of the following statements regarding laboratory studies of pleural fluid is true?
A. An elevated pleural fluid amylase level is uncommon in patients with a malignant pleural effusion
B. Pleural fluid eosinophilia is diagnostic of a pulmonary parasitic infection
C. A pleural liquid hematocrit that exceeds half the simultaneous peripheral blood hematocrit indicates frank bleeding into the pleural space and is diagnostic of a hemothorax
D. A pleural effusion with a pH of 5.8 is suggestive of empyema

Key Concept/Objective: To understand the clinical correlations of pleural fluid laboratory abnormalities

Determination of the pleural liquid amylase level is warranted in patients with unexplained left-sided pleural effusions, particularly in the presence of coexistent abdominal disease. Elevated amylase levels are also commonly seen in patients with malignancy. Pleural liquid eosinophilia is rarely the result of a fungal or parasitic infection. Much more commonly, the eosinophilia is a nonspecific finding; in some cases, it is thought to result from the previous introduction of air or blood into the pleural space. A pleural liquid hematocrit that exceeds half the simultaneous peripheral blood hematocrit indicates frank bleeding into the pleural space and is diagnostic of a hemothorax. In patients who have a pleural effusion associated with bacterial pneumonia (parapneumonic effusion), a pleural liquid pH of less than 7.0 is suggestive of an infected pleural space (empyema). A pH of 6.0 or less suggests esophageal rupture. (Answer: C—A pleural liquid hematocrit that exceeds half the simultaneous peripheral blood hematocrit indicates frank bleeding into the pleural space and is diagnostic of a hemothorax)

50. An 86-year-old man is admitted to the hospital for volume depletion and inability to care for himself.
He has experienced a 30-lb weight loss over the past several months, and his appetite has been poor. His family reports that his mental status has been normal. He has a long history of tobacco and alcohol abuse. Routine blood work reveals the following: iron deficiency anemia, a blood urea nitrogen/creatinine ratio of 25, and a mildly elevated alkaline phosphatase level. Other findings are normal. Chest xray reveals a moderate left pleural effusion. Diagnostic thoracentesis yields 60 ml of milky white fluid.

Which of the following statements regarding chylothorax is true?
A. Mediastinal tumors are the most common cause of chylothorax
B. There is no clinical correlation between lymphangiomyomatosis and chylothorax
C. In most cases of chylothorax, the triglyceride concentration exceeds 400 mg/dl
D. Chylous effusions will typically reaccumulate at the rate of 100 to 200 ml/day

Key ConceptObjective: To know the clinical features of chylothorax

Various conditions can cause chylothorax. Mediastinal tumors are the most common cause, with lymphomas exceeding metastatic carcinomas in frequency. Chylothorax frequently occurs as a complication of the rare disease lymphangiomyomatosis. In most cases of chylothorax, the triglyceride concentration exceeds 110 mg/dl; exceptions usually are limited to patients in whom feedings have been withheld, such as postoperative patients. The major consequence of chylous effusions is the rapid and recurrent accumulation of liquid in the pleural space. Normally, the thoracic duct transports chyle at a rate of 1.5 to 2.5 L/day. In patients with chylothorax, much or all of this liquid may enter the pleural space. (Answer: A—Mediastinal tumors are the most common cause of chylothorax)

51. A 65-year-old female smoker presents to your office for evaluation of weight loss and general malaise. Physical examination is remarkable for cachexia, dullness to percussion with associated decreased breath sounds, and decreased tactile fremitus over the left lung base. You order a chest radiograph, which reveals a left-sided pleural effusion. Subsequent lateral decubitus films reveal that the effusion is free-flowing. You perform a thoracentesis and send the fluid to the laboratory for analysis.

Which of the following findings would NOT be consistent with an exudative pleural effusion or one resulting from lymphatic obstruction?
A. A ratio of pleural protein to serum protein greater than 0.5
B. A ratio of pleural lactate dehydrogenase (LDH) to serum LDH greater than 0.6
C. A pleural LDH concentration greater than two thirds of the upper limit of normal for serum LDH
D. A pleural protein concentration of 1 g/dl
E. A pleural cholesterol level greater than 60 mg/dl

Key Concept/Objective: To know the criteria used to distinguish exudative pleural effusions from effusions resulting from lymphatic obstruction

If the pleural effusion does not appear macroscopically to be blood, pus, or chyle, then the diagnosis requires differentiating between an exudative process and a transudative process. Pleural effusions resulting from exudation or obstruction of lymphatic drainage typically have a protein concentration of 3 g/dl or greater. However, the following four criteria are more likely to correctly identify an exudative effusion: (1) a ratio of pleural protein to serum protein greater than 0.5; (2) a ratio of pleural LDH to serum LDH greater than 0.6; (3) a pleural LDH concentration greater than two thirds of the upper limit of normal for serum LDH; (4) a pleural cholesterol level greater than 60 mg/dl. The presence of any of these findings makes the diagnosis of an exudative effusion more likely. The absence of all four findings points toward a transudative effusion. (Answer: D—A pleural protein concentration of 1 g/dl)

52. A 60-year-old man suffers from chronic obstructive pulmonary disease (COPD). He is brought to see you urgently for chest pain and shortness of breath. He is obviously having difficulty breathing but is able to give some history. He states that he experienced the sudden onset of sharp right-sided chest pain just a few minutes before calling for an ambulance. The pain is associated with shortness of breath, but he denies having any wheezing, fever, chills, or cough. After examining the patient, you obtain a chest radiograph, which reveals a right-sided pneumothorax.

Which of the following physical examination findings is NOT consistent with the diagnosis of pneumothorax?
A. Hyperresonance to percussion on the affected side
B. Distant or absent breath sounds on the affected side
C. Expansion of the hemithorax on the affected side
D. Increased tactile fremitus on the affected side
E Diminished transmission of voice sounds on the affected side

Key Concept/Objective: To know the physical examination findings associated with pneumothorax

The patient with pneumothorax should have decreased tactile fremitus on the affected side. All of the other findings discussed in the case are consistent with the diagnosis of pneumothorax. This case illustrates the most common symptoms of pneumothorax: chest pain and dyspnea. In addition, this patient is a good example of someone in whom pneumothorax should be suspected, given his sudden onset of chest pain and dyspnea in conjunction with his history of chronic airflow obstruction. The diagnosis is confirmed with a chest radiograph. (Answer: D—Increased tactile fremitus on the affected side)

53. A 32-year-old man develops a nonproductive cough and experiences some decrease in his exertional tolerance, secondary to dyspnea, and general malaise. He works as a lawyer for a large corporation. During your evaluation, you order a chest radiograph, which reveals bilateral hilar adenopathy.

What is the most likely diagnosis for this patient?
A. Lymphoma
B. Metastatic cancer
C. Tuberculosis
D. Sarcoidosis
E. Berylliosis

Key Concept/Objective: To know the most common cause of bilateral hilar adenopathy and to know the differential diagnosis

The most common cause of bilateral hilar adenopathy is sarcoidosis, especially in those persons between 20 and 40 years of age. Lymphoma, tuberculosis, malignancy, and berylliosis should all be included as diagnostic possibilities. Lymphoma is often accompanied by lymphadenopathy at other sites, systemic symptoms, and anemia. When hilar adenopathy is a manifestation of metastatic disease, the primary malignancy is usually known or easily identifiable. Chronic granulomatous diseases such as tuberculosis and histoplasmosis usually present with unilateral rather than bilateral hilar adenopathy. It can be difficult to differentiate berylliosis from sarcoidosis; in the former, there is usually a history of occupational exposure to beryllium in the manufacture of alloys, ceramics, or high-technology electronics. (Answer: D—Sarcoidosis)

For more information, see Staton GW, Ingram RH Jr: 14 Respiratory Medicine: IX Disorders of the Pleura, Hila, and Mediastinum. ACP Medicine Online (www.acpmedicine.com). Dale DC, Federman DD, Eds. WebMD Inc., New York, February 2004

Pulmonary Edema

54. A 65-year-old man is admitted to the intensive care unit for mechanical ventilation. There are no family members available to discuss the patient’s history or current care. On arrival at the emergency department, the paramedics told the staff that the patient was “found down” in the park and smelled of alcohol. His initial hemoglobin oxygen saturation was 60%, and respirations were labored; thus, the patient was urgently intubated. Results of physical examination are as follows: temperature, 95.4° F (35.2° C); heart rate, 120 beats/min; respiratory rate, 20 breaths/min on mechanical ventilatation; and diffuse rales heard bilaterally in the lung fields. The patient is generally disheveled, with poor hygiene. Chest x-ray reveals bilateral interstitial and alveolar infiltrates. ECG reveals Q waves throughout the precordial leads.

Which of the following statements regarding the differentiation between cardiogenic and noncardiogenic pulmonary edema is true?
A. A bat’s-wing or butterfly pattern on chest x-ray is more typical of noncardiogenic than cardiogenic pulmonary edema
B. Distinct air bronchograms are more common with cardiogenic pulmonary edema
C. A widened vascular pedicle and an increase in the cardiothoracic ratio suggest cardiogenic pulmonary edema
D. Pulmonary arterial catheterization will yield useful information in these patients and will decrease their overall mortality

Key Concept/Objective: To know how to differentiate cardiogenic pulmonary edema from noncardiogenic pulmonary edema

Ancillary features that can be routinely visualized on an anteroposterior chest radiograph made with a portable x-ray machine may help differentiate cardiogenic from noncardiogenic pulmonary edema. A widened vascular pedicle and an increase in the cardiothoracic ratio suggest increased pulmonary capillary pressure; distinct air bronchograms are more common with noncardiogenic pulmonary edema. A predominantly perihilar distribution of pulmonary edema is common, and occasionally, there is a very sharp demarcation between the central area of pulmonary edema and the lung periphery, leading to a so-called bat’s-wing or butterfly pattern. This pattern is more typical of cardiogenic than noncardiogenic pulmonary edema. Despite the logical appeal of the use of pulmonary arterial catheters, no beneficial effect on outcome has been attributed to their use. A study of a large number of patients in intensive care units has suggested that patients who had pulmonary arterial catheters had a higher mortality at a higher financial cost than patients who did not undergo catheterization. (Answer: C— A widened vascular pedicle and an increase in the cardiothoracic ratio suggest cardiogenic pulmonary edema)

55. A 61-year-old woman presents to the emergency department. She was in her usual state of health until
2 days ago, when she developed fever; a cough productive of rusty sputum; chills; and exertional dyspnea. She denies having any contact with sick persons, and she has otherwise been very healthy. Results of physical examination are as follows: temperature, 102.3° F (39.4° C); heart rate, 105 beats/min; respiratory rate, 30 breaths/min; blood pressure, 80/42 mm Hg; hemoglobin oxygen saturation on room air,
84%; hyperdynamic precordium noted; jugular veins, flat; rales, heard diffusely in bilateral lung fields; and egophony noted, with increased fremitus in the right midlung zone. Laboratory data reveal leukocytosis with a left shift; thrombocytopenia; anemia with schistocytes; elevated coagulation parameters; mild renal insufficiency; and a moderate metabolic acidosis with an elevated anion gap. Chest x-ray reveals a diffuse alveolar filling process with focal consolidation of the right middle lobe. In the emergency department, the patient’s respiratory failure worsens, and she is urgently intubated.

Which of the following statements regarding the diagnosis and management of acute respiratory distress syndrome (ARDS) is true?
A. There are high levels of neutrophils and their secretory products in the bronchoalveolar lavage liquid of patients with ARDS
B. The alveolar filling process of ARDS affects all lung units equally
C. Gas exchange in ARDS is characterized initially by hypoxemia that is refractory to increasing concentrations of inspired oxygen, implying the presence of V/Q mismatching
D. Respiratory failure is the most common cause of death in patients with ARDS

Key Concept/Objective: To understand the diagnosis and management of ARDS

In ARDS patients, the underlying inflammatory response causes high levels of neutrophils and their secretory products in bronchoalveolar lavage liquid; this characteristic distinguishes noncardiogenic from cardiogenic edema. Typically, portable anteroposterior chest radiography reveals a diffuse and homogeneous alveolar filling process. When examined by CT, however, the air-space filling pattern frequently appears less homogeneous. Radiographs with the patient in the supine position typically show a greater degree of consolidation in posterior lung zones than in anterior lung zones. Gas exchange in ARDS is characterized initially by hypoxemia that is refractory to increasing concentrations of inspired oxygen, implying the presence of increased intrapulmonary shunting. Sepsis is the most common cause of death during the course of illness. As a result of state-of-the-art ventilatory-support techniques, respiratory failure is the cause of death in fewer than 20% of cases—a fact that highlights the importance of dysfunction of other organ systems in causing morbidity and mortality. (Answer: A— There are high levels of neutrophils and their secretory products in the bronchoalveolar lavage liquid of patients with ARDS)

56. A 65-year-old man presents to the emergency department complaining of progressive shortness of breath and lower leg swelling. Evaluation reveals increased jugular venous pressure, bilateral crackles, an S3 gallop, and moderate lower extremity edema. Arterial blood gas assessment reveals hypoxemia. A chest radiograph shows cardiomegaly and bilateral pulmonary edema.

What is the most common cause of cardiogenic pulmonary edema?
A. Renal failure
B. Left ventricular dysfunction
C. Mitral valve disease
D. Pulmonary venous obstruction

Key Concept/Objective: To understand the most common cause of cardiogenic pulmonary edema

Cardiogenic pulmonary edema is caused by increased capillary pressure (hydrostatic forces); fluid accumulates first in the airways, then in the alveolar interstitium, and finally in the alveolar space. The most common cause of cardiogenic pulmonary edema is left ventricular dysfunction. In congestive cardiomyopathy, the systolic performance of the left ventricle is impaired, the ventricle is dilated, and left ventricular end-diastolic pressure (LVEDP) is increased. The rise in LVEDP leads to an increase in pulmonary capillary pressure. (Answer: B—Left ventricular dysfunction)

57. The family of a 72-year-old female patient meets with you to discuss her condition. The patient was admitted to the intensive care unit with ARDS 2 days ago and has required mechanical ventilation and vasopressors. They ask you about her prognosis.

Which of the following is NOT a factor associated with a worse prognosis?
A. Age less than 65 years
B. Higher number of organ systems in failure
C. Higher number of days of organ-system failure
D. Longer duration of positive pressure ventilation

Key Concept/Objective: To know the factors associated with a worse prognosis for patients with ARDS

ARDS is frequently part of a systemic inflammatory response syndrome. This highlights the importance of multiple organ systems in the course of the disease. Factors associated with higher mortality in patients suffering from ARDS include a higher number of organ systems in failure, a higher number of days of organ-system failure, and age greater than 65 years. A longer duration of positive pressure ventilation is associated with a worse pulmonary functional outcome. The most common cause of death in patients with ARDS is sepsis. (Answer: A—Age less than 65 years)

For more information, see Staton GW, Ingram RH Jr: 14 Respiratory Medicine: X Pulmonary Edema. ACP Medicine Online (www.acpmedicine.com). Dale DC, Federman DD, Eds. WebMD Inc., New York, September 2004

Pulmonary Hypertension, Cor Pulmonale, and Primary Pulmonary Vascular Diseases

58. A 34-year-old woman with a diagnosis of primary pulmonary hypertension returns for evaluation. She has had a progressive increase in her dyspnea over the past 6 months. She has had marked fatigue for the past 4 months. Last month, she developed hoarseness. Evaluation by an otolaryngologist led to a diagnosis of Ortner syndrome. One week ago, she had onset of chest pain and an episode of syncope.

Of the symptoms this patient has had, which one suggests the worst prognosis?
A. Chest pain
B. Dyspnea
C. Fatigue
D. Syncope
E. Hoarseness

Key Concept/Objective: To know the symptoms of pulmonary hypertension and their prognostic significance

All of the symptoms listed are associated with pulmonary hypertension. Chest pain can mimic angina pectoris, and hoarseness can occur because of compression of the recurrent laryngeal nerve by enlarged pulmonary vessels (Ortner syndrome). Syncope and right heart failure generally occur later in the course of illness and are associated with a poorer prognosis. (Answer: D—Syncope)

59. A 32-year-old man comes to your office for a job-related injury. His family history is remarkable for two relatives who had “internal bleeding” in their 40s. On examination, you notice multiple small telangiectasias on his lips, skin, and oral mucosa. His fingernails appear slightly clubbed. Chest x-ray reveals several small, perfectly round nodules in both lungs.

Which of the following is the most accurate statement about this patient’s condition?
A. He is likely to develop pulmonary hypertension and right heart failure
B. Orthopnea is common in this disorder
C. He has an increased risk of stroke and brain abscess
D. His pulmonary function tests will show significant restrictive disease
E. There is no need to consider treatment if he remains asymptomatic

Key Concept/Objective: To be able to recognize hereditary hemorrhagic telangiectasia and to know its consequences

In this disorder, there are often numerous arteriovenous malformations (AVMs) in the lungs and elsewhere in the body. Such patients have an artificially low pulmonary resistance because a substantial fraction of blood may be shunting through the AVMs. Although the presence of AVMs generally does not lead directly to pulmonary hypertension, occasionally pulmonary hypertension is seen in association with AVM therapy; that is, if AVMs are resected, one can develop pulmonary hypertension because of vascular remodeling and an abrupt increase in resistance once the AVMs are no longer able to shunt blood. Orthopnea is actually unusual in this disorder; classically, patients have increased dyspnea when standing up, a symptom called platypnea. Pulmonary function tests are generally normal except for a slightly diminished diffusing capacity of lung for carbon monoxide (DLco). The long-term risk associated with the disease is largely the possibility that a clot or organism could embolize through one of these malformations directly to the brain. This makes treatment of asymptomatic patients controversial, but some favor it to prevent negative neurologic outcomes. (Answer: C—He has an increased risk of stroke and brain abscess)

60. Which of the following statements is true regarding primary pulmonary hypertension?

A. Right heart failure is a contraindication to lung transplantation
B. Calcium channel blockers are not effective therapy
C. Subcutaneous epoprostenol is a safe and effective treatment
D. Five-year survival is roughly similar with medical therapy and lung transplantation
E. Prognosis is excellent with early treatment

Key Concept/Objective: To understand the management of primary pulmonary hypertension

Primary pulmonary hypertension is a challenging and rare disease with a poor prognosis; 5-year survival is around 50% for both medical therapy and transplantation. Right heart failure often improves with a single-lung transplant and is not considered a contraindication to transplantation. Both calcium channel blockers and epoprostenol have been shown to be effective, and both can cause significant rebound pulmonary hypertension if stopped abruptly. Because of epoprostenol’s short half-life (3 to 6 minutes), it is delivered by continuous I.V. infusion, and complications from the pump or vascular access devices cause significant problems. (Answer: D—Five-year survival is roughly similar with medical therapy and lung transplantation)

For more information, see Staton GW, Ingram RH Jr: 14 Respiratory Medicine: XI Pulmonary Hypertension, Cor Pulmonale, and Primary Pulmonary Vascular Diseases. ACP Medicine Online (www.acpmedicine.com). Dale DC, Federman DD, Eds. WebMD Inc., New York, March 2003